PRAYAS - Geography Workbook - Questions - Compressed

Download as pdf or txt
Download as pdf or txt
You are on page 1of 108

PREFACE

Curiosity is fundamental to human existence and while preparing for one of the toughest exams,
one is naturally curious as to how to learn and practice for gateway (PRELIMS) to civil services
dream.
Thus, to satisfy this eagerness of an aspirant and help one prepare for UPSC prelims thoroughly
we have the best possible solution for you i.e. PRAYAS- PRACTICE WORK BOOK. It is
based on "reverse engineering technique" of analysing the syllabus of Geography in elementary
form.
PRAYAS constitutes prudent effort of various experts who have researched and analysed the
fundamental pattern of asking previous year questions. Covering any subject completely is
always a tedious task but PRAYAS has been framed with an innovative approach of completing
the syllabus comprehensively through 470+ MCQs along with solution. It will help candidates
to revise the concepts, facts and figures related to Geography, eventually catalysing their efforts
to clear arduous phase of prelims.
We sincerely hope, PRAYAS will serve as a valuable guide and companion for the aspirants
who are preparing for the Geography section of the UPSC prelims exam.
INDEX

1. SOLAR SYSTEM ......................................................................................................................................... 2


2. DISTRIBUTION OF CONTINENTS ................................................................................................................ 9
3. MINERALS AND ROCKS ........................................................................................................................... 14
4. LANDFORMS AND THEIR EVOLUTION ..................................................................................................... 22
5. GEOMORPHIC PROCESSES ...................................................................................................................... 29
6. EARTHQUAKE, VOLCANO........................................................................................................................ 34
7. RELIEF OF OCEAN .................................................................................................................................... 38
8. MOVEMENTS OF OCEAN ........................................................................................................................ 43
9. COMPOSITION OF ATMOSPHERE ............................................................................................................ 47
10. SOLAR RADIATION AND HEAT BALANCE ................................................................................................. 50
11. PLANETARY WINDS................................................................................................................................. 54
12. WIND CIRCULATION ............................................................................................................................... 62
13. WORLD CLIMATE .................................................................................................................................... 68
14. INDIA AND LOCATION............................................................................................................................. 72
15. DRAINAGE .............................................................................................................................................. 76
16. INDIAN CLIMATE..................................................................................................................................... 82
17. NATURAL VEGETATION........................................................................................................................... 88
18. AGRICULTURE ......................................................................................................................................... 93
19. MINERAL RESOURCES ........................................................................................................................... 100
20. MISCELLANEOUS .................................................................................................................................. 105

1
Solar system
Consider the following statements with respect
to universe
Consider the following statements:
1. Singularity in the Universe existed
1. Size
immediately after the big bang event.
2. Expanding universe hypothesis suggests the 2. Shape
expansion of individual galaxies. 3. Higher solar winds
3. Big bang theory explains the steady state of the
4. Gravity
universe at any point of time.
Which of the above factors are responsible for the
Which of the above statement are incorrect? gaseous nature of the Jovian Planets?
(a) 1 and 3 only (a) 1, 2 and 3 only
(b) 2 and 3 only
(b) 2, 3 and 4 only
(c) 1 and 2 only
(d) 1, 2 and 3 (c) 1, 3 and 4 only
(d) 1, 2, 3 and 4
Consider the following statements with
reference to Stars:
Which of the following correctly describes the
1. Stars are the localised lumps of Hydrogen and event's big splat?
Nitrogen within a galaxy.
(a) Collapse of a neutron star to a black hole.
2. A star can turn into a Black Hole only after
(b) Theory for the creation of the moon.
attaining a certain mass.
(c) Big bang event for the creation of the galaxy.
Which of the statements given above is/are correct? (d) Striking of meteorites on the surface of earth.
(a) 1 only
(b) 2 only
Consider the following statements:
(c) Both 1 and 2
(d) Neither 1 nor 2 1. Differentiation caused the separation of
Earth’s layers based on density.

Which is the most appropriate statement 2. Gases in the Earth’s atmospheric layer were
regarding our Solar system? stratified by the process of degassing.

(a) Light year is the amount of time taken by light 3. Life on Earth appeared first on the oceans
to cover a solar distance. rather than on land.

(b) The illumination of Planets is less than that of Which of the statements given above is/are correct?
stars in the sky. (a) 1 and 3 only
(c) Orion is the name given to the brightest star in (b) 3 only
the solar system.
(c) 1 and 2 only
(d) The period of revolution of a planet around a
(d) 2 and 3 only
star increases with the increase in distance.

2
Which of the statements given above are incorrect?
Consider the following statements with (a) 1 and 3 only
reference to the Gravitational Force:
(b) 2 and 3 only
1. Gravitational force provides a direct source of
(c) 1 and 2 only
information regarding Earth’s interior.
(d) None of the above
2. Gravitational force is more near the poles than
at the equator.
Consider the following Earthquake waves:
Which of the statements given above is/are correct?

(a) 1 only 1. P-waves

2. S-waves
(b) 2 only
3. Surface waves
(c) Both 1 and 2
What is the correct order of arrival of these waves
(d) Neither 1 nor 2
on the Earth’s surface?

(a) 3-2-1
Consider the following statements:
(b) 3-1-2
1. The interior of the Earth is constantly shaped
by the exogenic forces. (c) 1-2-3

2. The properties of Earth’s materials at (d) 1-3-2

different depths are governed by the


temperature and pressure. Which of the following statements is most
Which of the statements given above is/are correct? appropriate regarding the Earthquake waves?

(a) 1 only (a) P-waves are responsible for creating density


differences in the material they pass through.
(b) 2 only
(b) The shadow zone of S-wave is smaller as
(c) Both 1 and 2
compared to the P-waves.
(d) Neither 1 nor 2
(c) Body waves are the most damaging of all the
earthquake waves.
Consider the following statements regarding
(d) Most of the earthquake waves vibrate parallel
the Earthquakes:
to the direction of wave propagation.
1. The point where the energy of the Earthquake
is released inside the earth is called the
epicentre. Consider the following phenomenon:
1. Construction of large reservoirs
2. The direction of the Earthquake waves depends
2. Excessive mining activities
upon the density of materials it meets.
3. Gravitational force
3. Lesser the density of the material, more is the 4. Volcanic eruptions
velocity of the earthquake waves. 5. Strong tidal waves

3
Which of the above phenomena are responsible for Which of the following consequences would
the occurrence of the Earthquakes? follow if the axis of the Earth were
(a) 2, 3 and 4 only perpendicular to the plane of its elliptical orbit

(b) 1, 2 and 5 only of revolution?

(c) 1, 2 and 4 only (a) Varying lengths of day and night at different
(d) 1, 2, 3, 4 and 5 parts of the earth.

(b) Occurrence of the equinox at exactly the

Consider the following pairs: middle of the year.

(c) Equal duration of day and night at different


Phenomena Associated
parts of the earth.
layers
(d) None of the above.
1. Moho Discontinuity: Mantle- core

2. Gutenberg’s Crust- mantle


Consider the following statements regarding
Discontinuity:
the interior of the Earth:
3. Asthenosphere: Mantle only 1. The continents of the Earth are said to be floating
4. Lithosphere: Crust only on the oceanic crust.

Which of the above pairs are incorrectly matched? 2. The oceanic crust is made of the granitic rocks
while the continents are formed of basaltic
(a) 1, 2 and 4 only
rocks.
(b) 1, 2 and 3 only
Which of the statements given above is/are correct?
(c) 2, 3 and 4 only
(a) 1 only
(d) None of the above
(b) 2 only
(c) Both 1 and 2
Consider the following statements:
(d) Neither 1 nor 2
1. Shield volcanoes are the most explosive
volcanoes made up of basaltic lava.
2. Deccan Trap is an example of a composite Consider the following statements:
volcano largely composed of pyroclastic lava. 1. Igneous rocks are crystalline in structure and
may include some traces of fossils.
Which of the statements given above is/are correct?
2. Plutonic igneous rocks are the ones that are
(a) 1 only
cooled below the surface of the earth.
(b) 2 only
(c) Both 1 and 2 3. Marble, graphite, and schists are the important
(d) Neither 1 nor 2 examples of igneous rocks.
Which of the statements given above is/are correct?
(a) 1 and 3 only
(b) 2 only
(c) 2 and 3 only

4
(d) 3 only 2. Big Bang Theory considered the universe to
be roughly the same at any point of time.

Consider the following statements: Select the correct answer using the codes given
below:
1. Tropics in both the Hemisphere are the
maximum extent of the sun's motion. (a) 1 only

2. Obliques rays during winters are responsible (b) 2 only


for the lower temperature conditions. (c) Both 1 and 2
3. Periods of dawn and twilight are longer in the (d) Neither 1 nor 2
tropical latitudes.
4. June solstice is characterised by the shortest
Consider the following pairs:
day and longest nights in the Northern
Geological Time Major
Hemisphere.
‘Period’ event/Life
Which of the statements given above are correct?
1. Cretaceous Extinction of
(a) 1 and 3 only Period Dinosaurs
(b) 2 and 3 only 2. Tertiary Period Homo sapiens
(c) 1 and 2 only 3. Ordovician Period First Fish

(d) 3 and 4 only Which of the pairs given above is/are correctly
matched?
(a) 1 and 2 only
Which of the following is the most appropriate
regarding seasons on Earth? (b) 2 and 3 only

(a) Rotation of Earth around its axis is (c) 1 and 3 only


responsible for the occurrence of seasons. (d) 1,2 and 3
(b) Effect of seasonality is less pronounced in the
areas within the tropics.
On the planet earth, most of the freshwater
(c) Axial tilt of the Earth is responsible for the exists as ice caps and glaciers. Out of the
early arrival of spring in the Northern remaining freshwater, the largest proportion:
Hemisphere. (a) is found in atmosphere as moisture and clouds
(d) There is a positive heat balance during the (b) is found in freshwater lakes and rivers
December solstice in the Northern
(c) exists as groundwater
Hemisphere.
(d) exists as soil moisture

With reference to the theory of origin of Earth,


which of the following statements is/are Consider the following statements:
correct? 1. Earth has the highest density among all the
1. ‘Nebular Hypothesis’ which is also called planets.
‘expanding universe hypotheses was given 2. Jupiter has the highest gravity among all the
by Laplace. planets.

5
3. Mercury has the lowest escape velocity among (a) 1 only
all the planets. (b) 1 and 2 only
Which of the statements given above is/are correct? (c) 2 and 3 only
(a) 1 and 2 only
(d) 3 only
(b) 2 and 3 only
(c) 1 and 3 only Which of the following statements is/are the
(d) 1, 2 and 3 features of Sedimentary rocks?
1. Three- Fourth of the Earth’s surface is
Consider the following statements in context covered with these rocks.
of Oort Clouds: 2. These rocks are devoid of mineral resources.
1. It is the most distant region in our solar 3. These rocks are the source of some of the rich
system. soils.

2. It is a spherical layer of gaseous objects Select the correct answer using the codes given
surrounding our Sun, a star. below:

3. It is predicted that it is home to the long-term (a) 1 and 2 only


comets. (b) 1 and 3 only
Which of the statements given above is/are correct? (c) 2 and 3 only
(a) 1 and 2 only (d) 1,2 and 3
(b) 2 and 3 only
(c) 1 and 3 only Consider the following statements in context
(d) 1,2 and 3 of Earth’s Crust:
1. The Continental crust is thicker than the
oceanic crust.
Consider the following pairs:
2. The most abundant element of Earth’s crust
Layer of Earth Type of Discontinuity
is Silicon.
1. Upper Conrad discontinuity
Crust and 3. Average density of Crust is less than the
average density of the Earth.
Lower
Crust Which of the statements given above is/are correct?
2. Upper Moho-discontinuity (a) 1 and 2 only
Mantle (b) 2 and 3 only
and lower
(c) 1 and 3 only
Mantle
3. Outer Gutenberg (d) 1, 2 and 3

core and Discontinuity


inner core These volcanoes are characterised by
Which of the pairs given above is/are correctly eruptions of cooler and more viscous lavas
matched? than basalt. These volcanoes often result in
explosive eruptions. Along with lava, large

6
quantities of pyroclastic material and ashes 3. P-waves can travel through all three mediums
find their way to the ground. This material i.e., solid, liquid and gas whereas S-waves
accumulates in the vicinity of the vent can travel only through solid medium.
openings leading to formation of layers. Which
Select the correct answer using the codes given
of the following Volcanic forms is described in
below:
the above statement?
(a) Shield Volcano (a) 1 and 2 only

(b) Composite volcano (b) 2 only

(c) Caldera Volcano (c) 3 only

(d) Fissure type volcano (d) 1, 2 and 3

With reference to the difference between Match List 1 with List II and select the correct
acidic and basic rocks, which of the following answer using the code govern below the Lists:
statements is/are correct?
List I List II
1. Acidic rocks are characterised by high
content of silica as compared to basic rocks. (Active Volcano) (Name of country)

2. Acidic rocks have more content of heavy A. Taal volcano 1. Indonesia


metals as compared to basic rocks.
B. Sangay 2. New Zealand
3. Acidic soils are less resistant to weathering
Volcano
than basic soils.
Select the correct answer using the codes given C. White Island 3. Philippines
below: D. Mt. Merapi 4. Ecuador
(a) 1 only
Code:
(b) 1 and 2 only
(a) A-1, B-3, C-2, D-4
(c) 2 and 3 only
(b) A-4, B-3, C-1, D-2
(d) 3 only
(c) A-3, B-4, C-1, D-2

(d) A-3, B-4, C-2, D-1


Which of the following statements is/are
correct about the difference between P and S Consider the following statements:
body waves? 1. Extrusive Igneous rocks are fine grained,
1. P-waves are longitudinal waves analogous to whereas Intrusive Igneous rocks are coarse
the sound waves whereas S-waves are grained.
transverse waves analogous to the light 2. Extrusive igneous rocks are also called
waves. volcanic rocks whereas intrusive igneous
2. The shadow region of S-waves is more than rocks are also called plutonic rocks.
that of P-waves. 3. Igneous rocks are free from fossils but source
of minerals of great economic value.
Which of the statements given above is/are correct?

7
(a) 1 and 2 only

(b) 2 only

(c) 3 only

(d) 1, 2 and 3

With reference to the metamorphic rocks,


consider the following statements:
1. The dynamic metamorphism takes place
under the stress of temperature.
2. In case of dynamic metamorphism, shale gets
converted into slate.
Which of the statements given above is/are correct?
(a) 1 only
(b) 2 only
(c) Both 1 and 2
(d) Neither 1 nor 2

Answer Key
1. Ans: D 17. Ans: B
2. Ans: B 18. Ans: C
3. Ans: D 19. Ans: B
4. Ans: C 20. Ans: D
5. Ans: B 21. Ans: C
6. Ans: A 22. Ans: C
7. Ans: B 23. Ans: D
8. Ans: B 24. Ans: C
9. Ans: A 25. Ans: A
10. Ans: C 26. Ans: B
11. Ans: A 27. Ans: C
12. Ans: C 28. Ans: B
13. Ans: A 29. Ans: A
14. Ans: D 30. Ans: D
15. Ans: C 31. Ans: D
16. Ans: A 32. Ans: D
33. Ans: D

8
Distribution of continents
Q1. Consider the following pairs: 3. The ocean floor sediments are very thin as
Plates Nature/characteristics compared to the Continental floor.
1. Transform New crust is generated
Boundaries as the plates pull away 4. Constant eruptions at the crest of oceanic
from each other. ridges pushes the oceanic crust on either side.
2. Convergent The crust is destroyed as Select the correct answer using the codes below:
Boundaries one plate dives under
another. (a) 1 and 3 only
3. Divergent Create Rift Valleys and
(b) 1 and 2 only
Boundaries new oceans.
Which of the pairs given above is/are correctly (c) 1, 3 and 4 only
matched? (d) All of the above
(a) 1 and 2 only
(b) 2 and 3 only Q4. Consider the following:
1. Pole-fleeing force
(c) 1 and 3 only
2. Placer Deposits.
(d) 3 only 3. Gravity
4. Tidal force.
Q2. With respect to the Plate Tectonics, consider 5. Radioactive currents within earth.
the following statements: Which of the above might be responsible for the
1. The plates are not fixed and move tectonic movements?
horizontally over the asthenosphere.
(a) 1, 3, 4 and 5 only
2. The continental rocks are much younger than
(b) 1, 2, 4 and 5 only
the ocean crust rocks.
(c) 1, 2, 3, and 5 only
3. The mid-oceanic ridges are most active in
terms of volcanic eruptions. (d) 1, 2, 3 ,4 and 5

Which of the statements given above is/are


incorrect? Q5. Consider the following statements given
below:
(a) 2 only
1. Continents are a part of a plate.
(b) 1 and 3 only 2. Mid-ocean ridges and Seafloor spreading
(c) 3 only doesn’t have any influence on the sea levels.
(d) 1 and 2 only 3. Seafloor spreading proves that the ocean
itself is a site of tectonic activity.
Which of the statements given above is/ are
Q3. Which of the following might suggest the
phenomenon of Sea floor Spreading: correct?
1. The age of the rocks increases as one moves (a) 1 and 2 only
away from the crest. (b) 2 and 3 only
2. Plates moving horizontally over the (c) 1 and 3 only
asthenosphere.
(d) All of the above.

9
Q6. Consider the following statements about the (c) Both 1 and 2
Sea floor spreading: (d) Neither 1 nor 2
1. Seafloor spreading is consistent at all mid-
ocean ridges.
Q9. Consider the following statements with respect
2. Seafloor spreading is the result of mantle to sea floor spreading hypothesis:
convection. 1. This hypothesis was coined by Arthur
Which of the statements given above is/are Holmes.
incorrect? 2. Rocks closer to the mid oceanic ridge have
(a) 1 only normal polarity and are the youngest.

(b) 2 only 3. Ocean crust rocks are much younger than the
continental rocks.
(c) Both 1 and 2
Which of the statements given above is/are correct?
(d) Neither 1 nor 2
(a) 1 and 2 only
(b) 2 and 3 only
Q7. In context of the Plate Movements, consider
the following statements: (c) 1,2 and 3
1. The rate of Plate Movements is uniform (d) 1 and 3 only
throughout.
2. Seafloor spreading occurs at convergent plate Q10. In the context of Hotspots, consider the
boundaries following statements:
3. The magnetic history of the earth is recorded
1. Hotspots develop near the Plate Boundaries.
in the spreading ocean floors.
2. Hot spots can be used to track plate
Which of the statements given above is/are correct? movements.
(a) 1 and 2 only Which of the statements given above is/ are
(b) 3 only correct?

(c) 2 and 3 only (a) 1 and 2 only

(d) 1 and 3 only (b) 1 only


(c) 2 only
Q8. With reference to the Plate tectonics, consider (d) Neither 1 nor 2
the following statements:
1. Convection current in mantle is one of the Q11. In the context of Distribution of Continents
driving forces behind the movement of the and oceans, consider the following statements:
plate.
1. The locations of oceans and land masses
2. Shallow focus earthquakes are usually of
influences the Earth system.
large spread, causing greater damage at the
surface or the earth's crust. 2. The basaltic magma erupts out of the Mid-
Oceanic Ridges.
Which of the statements given above is/are correct?
Which of the statements given above is/are correct?
(a) 1 only
(a) 1 only
(b) 2 only

10
(b) 2 only 2. Deccan traps were formed due to volcanic
(c) Both 1 and 2 eruptions after the Himalayan Mountain
building was started.
(d) Neither 1 nor 2
3. Indian plate boundaries include both
convergent and divergent boundaries.
Q12. With reference to the Placer Deposits, Which of the statements given above is/are
consider the following statements: incorrect?
1. Placer Deposits can be found in the rivers. (a) 1 and 2 only
2. Minerals that form placer deposits are (b) 1 and 3 only
chemically resistant to weathering.
(c) 2 and 3 only
3. Placer Deposits are one of the pieces of
(d) 1, 2 and 3
evidence in the support of the Sea Floor
spreading theory.
Q15. In the context of the distribution of continents
Which of the statements given above is/are correct?
and oceans, consider the following statements:
(a) 1 and 2 only
1. The Mid-Atlantic Ridge runs through
(b) 2 and 3 only Iceland.
(c) 1 and 3 only 2. Japan is situated along the Pacific Ring of
(d) All of the above. Fire.
3. The East African Rift Valley is a developing
convergent plate boundary.
Q13. In the context of the Plate Tectonics, consider
the following statements: Which of the statements given above is/are correct?
1. The plate boundaries influence the (a) 3 only
distribution of natural resources. (b) 1 and 2 only
2. Gold deposits are found in the rocks of the (c) 2 and 3 only
active volcanoes.
(d) 1 and 3 only
Which of the statements given above is/are
incorrect?
Q16. With reference to the Continental Drift
(a) 1 only
theory, Consider the following statements:
(b) 2 only
1. Continental Drift theory was propounded by
(c) Both 1 and 2 Alfred Wegener which suggests that
(d) Neither 1 nor 2 continents shift position on Earth's surface.
2. Gravitational force was considered as one of
the forces behind Continental Drift theory by
Q14. Consider the following statements with respect
to Indian plate: Wegener.
1. Eurasian plate has been sinking below the Which of the statements given above is/are correct?
Indian plate to form the Himalayas.
(a) 1 only
(b) 2 only

11
(c) Both 1 and 2 Q19. Arrange the following sections of the ocean
(d) Neither 1 nor 2 relief as they occur from the coast to the deep
sea:
1. Continental Slope
Q17. Which of the following pairs is /are correctly
matched? 2. Continental Rise
Minor Plates Location
3. Abyssal Plain
1 Cocos Plate Between South
America and 4. Continental Shelf
Pacific Plate
2 Fuji Plate North-east of Select the correct answer using the code given
Australia below.
3 Caroline Plate Between the (a) 4-2-1-3
Philippine and
Indian Plates (b) 4-1-2-3
Select the correct answer using the code given (c) 2-4-1-3
below:
(d) 2-1-4-3
(a) 1 and 2 only
(b) 2 and 3 only
Q20. Consider the following statements:
(c) 1 and 3 only
1. Mid-Atlantic ridge is an example of
(d) 1, 2 and 3 Constructive Plate Margins
2. Himalayas are formed due to convergence
Q18. Which of the following plates is/are not a part between oceanic and continental plates.
of the major tectonic plates of Earth’s
3. San Andreas Fault near California is the
lithosphere?
result of Destructive Plate Margins.
1. Pacific Plate
Which of the statements given above is/are correct?
2. Philippine Plate
(a) 1 only
3. Antarctica plate adjacent oceanic plate
(b) 1 and 2 only
4. Arabian plate
(c) 2 and 3 only
5. Eurasia and the adjacent oceanic plate
(d) 2 only
Select the correct answer using the code given
below.
Q21. Which of the following pairs is /are correctly
(a) 1 and 2 only matched
(b) 2 and 4 only Ocean Characteristics
Relief
(c) 2 and 3 only 1 Guyot Flat-topped
(d) 4 and 5 only seamounts
2 Mid Chains of Mountains
oceanic at the sea-bottom
Ridges
3 Trenches Narrow and steep-
sided depressions

12
Select the correct answer using the code given
below:
(a) 1 and 2 only
(b) 2 and 3 only
(c) 1 and 3 only
(d) 1, 2 and 3

Q22. Which of the following is/are the evidence in


support of the Continental Drift theory?
1. Rich placer deposits of Gold in Ghana.
2. Gondwana system of tillilite sediments found
in landmasses of Northern Hemisphere.
3. Rock of the same age across the Oceans.
Select the correct answer using the code given
below:
(a) 1 only
(b) 1 and 3 only
(c) 1, 2 and 3
(d) 1 and 2 only

Answer Key
1. Ans: B 12. Ans: A
2. Ans: A 13. Ans: D
3. Ans: C 14. Ans: A
4. Ans: A 15. Ans: B
5. Ans: C 16. Ans: A
6. Ans: A 17. Ans: B
7. Ans: B 18. Ans: B
8. Ans: C 19. Ans: B
9. Ans: B 20. Ans: A
10. Ans: C 21. Ans: D
11. Ans: C 22. Ans: B

13
Minerals and rocks
Q1. Consider the following statements in the Which of the statements given above is/are correct?
context of Minerals and Rocks:
(a) 1 and 2 only
1. A rock is an aggregate of one or more
minerals. (b) 2 and 3 only

2. Minerals are solely formed by inorganic (c) 1 and 3 only


processes. (d) All of the above.
3. Divergent boundaries on continental crust
can create mineral deposits.
Q4. In the context of Rocks, consider the following
Which of the statements given above is/are correct? statements:

(a) 1 and 2 only 1. Plutonic Rocks are formed by the sediment


deposition over a period of time.
(b) 2 and 3 only
2. Many mountains are made out of igneous
(c) 1 and 3 only rocks.
(d) All of the above. 3. Extrusive igneous rocks are formed by the
cooling of molten magma on the earth's
Q2. In the context of Mineral distribution, surface.
consider the following statements:
Which of the statements given above is/are
1. Tectonic processes help in understanding the incorrect?
distribution of mineral deposits.
(a) 1 and 3 only
2. The North Eastern Peninsular Belt is one of
(b) 2 and 3 only
the poorest mineral belts of India.
(c) 1 only
3. Magnetite is the best iron ore found with
about 70 per cent metallic content. (d) 2 only

Which of the statements given above is/are correct?


(a) 1 and 2 only Q5. Consider the following statements about the
Sedimentary and Metamorphic Rocks:
(b) 2 and 3 only 1. Metamorphic rocks are formed from heat and
(c) 1 only pressure and sedimentary rocks are formed
(d) 3 only from compaction and cementation.
2. Metamorphic rocks contain fossilsWhereas
Sedimentary rocks are devoid of the same.
Q3. Consider the following statements about the
Rock cycle: Which of the statements given above is/are correct?
1. Crystallization and Sedimentation are some (a) 1 only
of the key processes of the Rock cycle.
(b) 2 only
2. Igneous rocks can be changed into
(c) Both 1 and 2
metamorphic rocks.
(d) Neither 1 nor 2
3. The rock cycle is temperature and pressure
dependent.

14
Q9. In the Context of Rocks of India, consider the
following statements:
Q6. Consider the following pairs,Which of the
following is/are correctly matched? 1. Dharwar System rocks are metamorphosed
rocks.
Associated Processes Rocks
1. Lithification Igneous Rocks 2. Cuddapah System rocks have been formed as
2. Recrystallization Metamorphic a result of erosion and sedimentation of the
rocks of Dharwar system.
Rocks
3. Crystallization Sedimentary 3. Archean rocks are the oldest rocks of the
Rocks. earth's crust.

Select the correct answer using the codes below: Which of the statements given above is/are correct?

(a) 1 and 3 only (a) 1 and 2 only

(b) 2 and 3 only (b) 2 and 3 only

(c) 2 only (c) 1 and 3 only

(d) 3 only (d) All of the above.

Q7. Consider the following statements about the Q10. With respect to the Minerals of India, consider
Mineral distribution of India: the following statements:

1. India is the largest exporter of Iron ore in the 1. The Jaduguda Mines of Jharkhand is the
world. largest Iron ore Mine of India.

2. Karnataka has very rich deposits of high- 2. India has more than 50 per cent of the world’s
grade iron ores. Uranium reserves.

Which of the statements given above is/are correct? Which of the statements given above is/are correct?

(a) 1 only (a) 1 only

(b) 2 only (b) 2 only

(c) Both 1 and 2 (c) Both 1 and 2

(d) Neither 1 nor 2 (d) Neither 1 nor 2

Q8. With respect to Metamorphic Rocks, consider Q11. Consider the following Pairs:
the following statements: Rocks Types
1. Metamorphic rocks are formed during plate 1. Shale Sedimentary Rock
tectonic processes. 2. Quartzite Igneous Rock
3. Schist Metamorphic Rock
2. Metamorphic Rocks are the basis of the Rock
Cycle. Which of the pairs given above is/are correctly
3. Dynamic metamorphism occurs along the matched?
fault zones. (a) 1 and 2 only
Which of the statements given above is/are correct? (b) 1 and 3 only
(a) 1 only (c) 2 and 3 only
(b) 1 and 3 only (d) 1 only
(c) 2 and 3 only
(d) 1 and 2 only

15
Q12. With reference to different types of rocks, Q15. With respect to Bauxite, consider the following
consider the following statements: statements:
1. Foliation is the salient feature of Sedimentary 1. Bauxite is a sedimentary rock.
rocks. 2. Telangana is the largest Bauxite producer in
2. Sedimentary rocks serve as a record of the India.
events and processes that shaped the surface 3. The Bayer process is a method of refining
of Earth. bauxite to produce alumina.
3. Extrusive Igneous Rocks are characterised by Which of the statements given above is/are
their large grain textures. incorrect?
Which of the statements given above is/are (a) 2 only
incorrect?
(b) 1 and 3 only
(a) 1 and 2 only
(c) 2 and 3 only
(b) 2 only
(d) 3 only
(c) 1 and 3 only
(d) 3 only
Q16. Consider the following statements:
1. Sulphur, Aluminum and nitrates are
Q13. Consider the following: examples of non-metallic minerals.
1. The mineral composition of the parent rock. 2. Rajasthan has the largest reserves of copper
2. Amount of pressure applied. ore in India.
3. Presence of Water. Which of the statements given above is/are correct?
4. Temperature Stress. (a) 1 only
Which of the above might be the responsible (b) 2 only
factors that control metamorphic processes? (c) Both 1 and 2
(a) 1, 2 and 4 only (d) Neither 1 nor 2
(b) 2, 3 and 4 only
(c) 2 and 4 only Q17. Consider the following about the Sedimentary
(d) All of the above. Rocks:
1. Sedimentary rocks have layers because of
Q14. In the context of Minerals and Rocks, consider continuous outpouring of lava in layers.
the following statements: 2. These rocks can be a source of major energy
1. Basalt rocks are created at mid-ocean ridges. resources.

2. Metamorphism may change the mineral 3. Sedimentary rocks tend to be more porous
composition and the texture of the rock. than igneous rocks.

3. The most common igneous rock found at the Which of the statements given above is/are correct?
earth's surface is Marble. (a) 1 only
Which of the statements given above are correct? (b) 2 only
(a) 1 and 3 only (c) 2 and 3 only
(b) 1 and 2 only (d) All of the above
(c) 2 and 3 only
(d) All of the above.

16
Q18. In the context of Mica, consider the following Q21. Consider the following pairs with reference to
statements: characteristics of minerals:
1. Mica is useful as an electric insulator as it
does not conduct electricity.
2. No Mica reserves are found in the state of Characteristics Minerals
Andhra Pradesh.
3. Mica is widely used in cosmetics and makeup 1. Used in a) Olivine
products. ceramics and
Which of the statements given above are correct? glass

(a) 1 and 2 only 2. Found in b) Feldspar


(b) 2 and 3 only meteorites
(c) 1 and 3 only 3. Used in c) Pyroxene
(d) All of the above. jewellery

4. Used in d) Mica
Q19. With reference to the similarities between the Electrical
Metamorphic and Igneous Rocks, consider the
instruments
following statements:
1. Both igneous rock and metamorphic rocks
Which one of the options given below is correct?
are created under very hot temperatures.
(a) 1-a, 2-b, 3-d, 4-c
2. Both are formed by the cooling and
crystallization of the magma. (b) 1-b, 2-c, 3-a, 4-d
Which of the statements given above is/ are (c) 1-c, 2-b, 3-a, 4-d
correct? (d) 1-c, 2-d, 3-b, 4-a
(a) 1 only
(b) 2 only Q22. With reference to the igneous rocks, consider
(c) Both 1 and 2 the following statements:
1. The most fundamental properties of igneous
(d) Neither 1 nor 2
rocks are crystallinity and granularity.
2. Rocks formed by sudden cooling of molten
Q20. Consider the following statements about the material at the surface are called extrusive
Igneous Rocks: igneous rocks.
1. Sills are sheet-like igneous intrusions that cut 3. Granite is an example of intrusive igneous
across any layers in the rock they intrude. rocks.
2. Stratification occurs only in the Sedimentary 4. Basalt is not an example of extrusive igneous
Rocks. rocks.
3. Dikes are igneous rocks that intrude Which of the statements given above is/are correct?
vertically across the rocks.
(a) 3 only
Which of the statements given above is/are correct?
(b) 1, 2, 4 only
(a) 1 and 2 only
(c) 1, 2, 3 only
(b) 2 and 3 only
(d) 2 and 3 only
(c) 1 and 3 only
(d) 3 only

17
Q23. Consider the following statements with Which of the statements given above is/are
reference to the sedimentary rocks: incorrect?
1. Sedimentary rocks are formed by
(a) 1 only
consolidation and compaction of sediments.
(b) 2 only
2. Arenaceous sedimentary rocks have more
clay. (c) Both 1 and 2

3. Sedimentary rocks are fossiliferous. (d) Neither 1 nor 2

4. Chalk and coal are examples of organic


sedimentary rocks. Q26. Consider the following statements with
reference to the phenomenon of ‘Rock Cycle’:
Which of the statements given above is/are correct?
1. Igneous rocks, sedimentary rocks and
(a) 1 and 2 only metamorphic rocks after weathering, erosion
(b) 2 and 3 only and deposition can convert into sediments.

(c) 1, 2 and 3 only 2. Igneous rocks and sedimentary rocks on


applying heat and pressure can convert into
(d) 1, 3 and 4 only
metamorphic rocks.
3. Sediment on compaction and cementation
Q24. Which one of the following Rocks is formed by
converts into sedimentary rocks.
the processes given below?
1. The already consolidated rocks undergo Which of the statements given above is/are correct?
recrystallization and reorganization of (a) 1 and 2 only
materials within the original rocks.
(b) 2 and 3 only
2. The rocks exhibit lineation.
(c) 1 and 3 only
3. The rocks display banding.
(d) All of the above
Select the correct answer using the options given
below:
Q27. Consider the following statements with
(a) Sedimentary rocks reference to endogenic geomorphic processes:
(b) Extrusive igneous rocks 1. It is mostly generated by mass movement,
erosion, deposition and weathering.
(c) Metamorphic rocks
2. Continent building, earthquakes and plate
(d) Intrusive igneous rocks
tectonics are some examples of endogenic
processes.
Q25. Consider the following statements with
3. Actions of the endogenic forces are uniform.
reference to the metamorphism process:
1. Some rocks in the process of metamorphism Which of the statements given above is/are
get arranged in layers or lines. Such an incorrect?
arrangement is called banding. (a) 2 only
2. Sometimes minerals of different groups are (b) 3 only
arranged to alternating thin to thick layers
(c) 1 and 3 only
appearing in light and dark shade is called
foliation. (d) 2 and 3 only

18
Q28. Consider the following statements with (b) 1 and 3 only
reference to the weathering processes of rocks: (c) 2 and 3 only
1. Weathering is an ex-situ process and an (d) All of above
important process in the formation of soils.
2. Weathering aids mass wasting, erosion,
reduction of relief and changes in landforms. Q31. Consider the following statements with
reference to the physical weathering processes
3. Exfoliation is one of the effects of of rocks:
weathering.
1. Physical weathering processes are caused by
4. Weathering of rocks and deposits helps in the thermal expansion and pressure release.
enrichment and concentrations of certain
valuable ores of manganese, aluminum, iron 2. Physical weathering processes depend on
and copper. gravitational forces and water pressures also.
3. Frost wedging is an example of the physical
Which of the statements given above is/are correct?
weathering processes.
(a) 1 only
Which of the statements given above is/are correct?
(b) 2 and 3 only
(a) 1 and 2 only
(c) 1, 2 and 3 only
(b) 2 and 3 only
(d) 2, 3 and 4 only
(c) 1, 3 and 4 only
(d) All of above
Q29. Consider the following statements with
reference to the chemical weathering process
of ‘Solution’: Q32. Consider the following statements with
1. Solution is the most potent weathering reference to the exogenic geomorphic process
process in limestone. of ‘Mass Movement’:

2. Minerals like nitrate, sulphate and potassium 1. Mass movement is independent of the
are not affected by the solution process. influence of gravity.
2. Indiscriminate removal of natural vegetation
3. Sodium chloride is a rock-forming mineral
and is susceptible to the process of solution. is an activating cause of mass movement.

Which of the statements given above is/are correct? 3. The process of Mass movement comes under
erosion.
(a) 1 only
4. Weathering is a prerequisite for mass
(b) 2 only movement.
(c) 1 and 3 only Which of the statements given above is/are
(d) None of above incorrect?
(a) 1 and 3 only
Q30. Consider the following statements with (b) 1, 2 and 4 only
reference to the chemical weathering process (c) 1, 3 and 4 only
of ‘Hydration’:
(d) All of above
1. Hydration is an irreversible and long process.
2. Continued repetition of this process causes
fatigue in the rocks and may lead to Q33. Consider the following statements with
disintegration. reference to the rapid mass movements of
geomorphic processes:
3. Many clay minerals swell and contract during
wetting and drying. 1. Earth flow and Solifluction are the types of
rapid mass movement.
Which of the statements given above is/are correct?
(a) 1 only

19
2. Debris avalanche is much faster than Which of the statements given above is/are correct?
mudflow. (a) 1 only
3. Thick layers of weathered materials get (b) 3 only
saturated with water and flow down along
definite channels like a stream of mud is (c) 1 and 2 only
called mudflow. (d) 1, 2 and 3 only
Which of the statements given above is/are
incorrect? Q36. Consider the following statements with
reference to the Soil Genesis:
(a) 1 only
1. Humus content in soils plays a very important
(b) 2 only
role in its fertility.
(c) 2 and 3 only
2. Soil functions as a fundamental
(d) All of the above interfaceWhere the atmosphere, lithosphere,
hydrosphere, and biosphere meet.

Q34. Consider the following statements with 3. Soil is formed by the continuous weathering
reference to various geomorphic processes: of mountains over thousands of years.
1. Geomorphic processes and geomorphic Which of the statements given above is/are correct?
agents are the same unless stated separately.
(a) 1 and 2
2. Gravitational stresses are part of geomorphic
(b) 2 only
processes.
(c) 1 and 3 only
3. Diastrophism and volcanism are endogenic
geomorphic processes. (d) All of the above
4. Weathering, mass wasting, erosion and
deposition are exogenic geomorphic Q37. Which one of the following factors is incorrect
processes. regarding the formation of soils?
Which of the statements given above are correct? (a) Parental material

(a) 1 and 2 only (b) Topography and weathering

(b) 2 and 3 only (c) Climate

(c) 1 and 3 only (d) Soil texture

(d) All of the above


Q38. Consider the following statements with
reference to exogenic geomorphic processes:
Q35. With reference to the landslide as a 1. Mass wasting, erosion, deposition and
geomorphic activity, Consider the following
statements: weathering are exogenic geomorphic
processes.
1. Landslides are slow and perceptible
movements. 2. The exogenic geomorphic processes are
uniform across the regions.
2. Rapid rolling or sliding of earth debris with
backward rotation is called a debris slide. 3. The exogenic geomorphic processes are
mainly land building processes.
3. Slumps, rockslides and rockfalls are all the
types of landslides. Which of the given statements above is/are correct?

20
(a) 1 only (d) All of the above
(b) 1 and 2 only Q41. Consider the following statements with
reference to endogenic geomorphic processes
(c) 2 and 3 only of ‘Diastrophism’:
(d) All of the above 1. In orogenic processes, there is plate tectonics
involving horizontal movements of crustal
plates.
Q39. With reference to the erosional denudational
processes, Consider the following statements: 2. There can be faulting and fracturing of earth's
1. It acts physically but not chemically. crust in diastrophism.

2. Erosional agents include running water, 3. Epeirogenic is a continental building


wind, waves and currents, glaciers and ice. process.

3. Solar radiation is the ultimate source of Which of the statements given above is/are correct?
energy for these processes. (a) 1 and 2 only
4. Globally about 33% of the incoming (b) 1 and 3 only
radiation drives the hydrological cycle.
(c) 2 and 3 only
Which of the statements given above is/are correct?
(d) All of the above
(a) 1 only
(b) 1 and 2 only
Answer Key
(c) 2 and 3 only 1. Ans: C 22. Ans: C
(d) 2, 3 and 4 only 2. Ans: C 23. Ans: D
3. Ans: D 24. Ans: C
4. Ans: C 25. Ans: C
Q40. Consider the following statements with 5. Ans: A 26. Ans: D
reference to the Himalayas and the Western 6. Ans: C 27. Ans: C
Ghats: 7. Ans: B 28. Ans: D
1. The Himalayas mountain belt comprises 8. Ans: B 29. Ans: C
9. Ans: D 30. Ans: C
tectonically unstable younger geological
10. Ans: D 31. Ans: D
formations subjected to severe seismic 11. Ans: B 32. Ans: C
activity. 12. Ans: C 33. Ans: A
13. Ans: D 34. Ans: D
2. The Western Ghats and nilgiris are
14. Ans: B 35. Ans: B
geologically stable but have uplifted plateau 15. Ans: A 36. Ans: D
margins influenced by neo-tectonic activity. 16. Ans: B 37. Ans: D
3. Compared to Western Ghats region, the 17. Ans: C 38. Ans: A
18. Ans: C 39. Ans: D
slides in the Himalayas region are huge and
19. Ans: A 40. Ans: D
massive. 20. Ans: C 41. Ans: C
Which of the statements given above is/are correct? 21. Ans: B

(a) 1 only
(b) 1 and 2 only
(c) 2 and 3 only

21
Landforms and their evolution

Q1. Which of the following conditions are 4. Underlying structure of the rock
necessary for formation of karst topography? 5. Long term changes in the climate
1. Subsurface strata made up of impermeable Which of the above factors contribute to the
water-soluble rocks. evolution of a landform?
2. Moderate to abundant rainfall for chemical (a) 1, 2, 3 and 4 only
weathering.
(b) 2, 3 and 5 only
3. Thinly bedded and highly fractured and
(c) 1, 4 and 5 only
cracked rock strata.
(d) All of the above
4. A very high level of groundwater table.
Select the correct answer using the code given
below: Q4. Which of the following features are typically
associated with the process of Glaciation?
(a) 1,2 and 3 only
1. U-shaped glacial troughs
(b) 2 and 3 only
2. Hanging Valleys
(c) 1,2 and 4 only
3. Plucking of individual pieces of bedrock
(d) 3 and 4 only
4. Large eroded hills called Inselbergs
Choose the correct answer using the code given
Q2. Consider the following fluvial landforms:
below:
1. Potholes
(a) 1 and 2 only
2. Alluvial Fans
(b) 2,3 and 4 only
3. Paired River terraces
(c) 1,2 and 3
4. Deltas
(d) 1,3 and 4 only
Which of the above landforms are depositional in
nature?
Q5. Consider the following features:
(a) 1 and 2 only
1. Creeks
(b) 1, 3 and 4 only
2. Cirque
(c) 2 and 4 only
3. Barchans
(d) 3 and 4 only
4. Ox-Bow Lake
If you are travelling from the Gangotri Glacier to
Q3. Consider the following:
the Arabian Sea via the Thar desert, inWhich of the
1. Overland flow of water following orders will you encounter the above
2. Vegetation cover features?
3. Annual variation in temperature (a) 2-3-4-1

22
(b) 2-4-1-3
(c) 2-4-3-1 Q9. With respect to Coral reefs, consider the
following statements:
(d) 3-2-4-1
1. While barrier reefs grow near the coastline
around islands and continents, Fringing
Q6. Which of the following countries would you Reefs are separated from the continent by
travel to in order to see ‘Eskers’ and ‘Roche deeper and wider lagoons.
moutonnée’? 2. Atolls are usually located in the middle of the
(a) Congo sea.
(b) Scotland 3. Coral reefs are generally absent in the
(c) Mauritius Western margins of continents.

(d) Yemen Which of the above statements is/are correct?


(a) 1 and 2 only

Q7. Consider the following processes of erosion (b) 2 and 3 only


and landform development: (c) 2 only
1. Attrition (d) 1 and 3 only
2. Ablation
3. Deflation Q10. In the context of geomorphology, ‘Tombolos’
4. Abrasion and ‘Sand Spits’ are:

Which of the above are common to both Aeolian (a) Depositional landforms formed by wind
and Fluvial erosion cycles? action.

(a) 1 and 3 only (b) Erosional landforms formed by action of


waves and currents.
(b) 1 and 4 only
(c) Erosional landforms formed wind action.
(c) 1,2 and 4 only
(d) Depositional landforms formed by actions of
(d) 2,3 and 4 only waves and currents.

Q8. Consider the following statements: Q11. Consider the following statements:
1. Some rivers in India are antecedent in nature. 1. Tides refer to the rise and fall of sea levels
2. All stalagmites found naturally in India are caused by the combined effects of the
located in Meghalaya. gravitational forces exerted the Sun and the
3. Some portion of the Indian coastline is now Earth only.
submerged. 2. A rip current is a strong current of
4. All volcanoes in India are now dormant. waterWhich moves directly towards the
shore.
Which of the above statements are correct?
3. The formation of a bay takes placeWhere the
(a) 1 and 3 only
coast is made of alternate bands of hard and
(b) 1,2 and 4 only soft rocks.
(c) 1, 3 and 4 only Which of the above statements is/are correct?
(d) 2 and 3 only (a) 1 and 2 only

23
(b) 2 only (b) 2 and 3 only
(c) 2 and 3 only (c) 1,3 and 4
(d) 3 only (d) 1,2,3 and 4

Q12. With reference to various drainage patterns, Q15. Consider the following statements:
consider the following statements: 1. Block mountains often result from rifting and
1. In India, a Centripetal drainage pattern is faulting.
seen in the highlands of Amarkantak. 2. Fold mountains are createdWhere two or more of
2. In India, a Radial drainage pattern is seen in Earth's tectonic plates are pushed together due to
the Ladakh highlands. tension induced forces.
3. In India, no river flows through a rift valley. 3. A horst represents a block pushed upward relative
Which of the above statements is/are correct? to the blocks on either side by the faulting.

(a) 1 only Which of the above statements are correct?

(b) 2 only (a) 1 and 2 only

(c) 1 and 3 only (b) 2 and 3 only

(d) None of the above (c) 1 and 3 only


(d) 1, 2 and 3

Q13. “This landform is characterised by dry,


vegetation-free, flat bottom depression at the Q16. Where would you find “Gloups”, “stacks” and
lowest part of a desert basin. It is periodically “stumps”?
covered by water that slowly filtrates into the (a) In the Limestone topography of Andhra
groundwater system or evaporates into the Pradesh.
atmosphere.”
(b) Somewhere near the coast of Andaman and
The landform in the above excerpt is: Nicobar Islands.
(a) Hamada (c) Towards the border between India and
(b) Playa Pakistan across the Thar desert.
(c) Zeugen (d) Near the Kalapani region of the Indo-Nepal
(d) Yardang border.
Q17. Which of the following can be classified as an
intrusive landform?
Q14. Which of the following processes signify
Diastrophism: 1. Batholith

1. Orogenic Forces 2. Sill

2. Epeirogenic Forces 3. Dykes

3. Earthquakes 4. Lopolith

4. Plate Tectonics Select the correct answer using the code given
below.
Select the correct answer using the code given
below: (a) 1 and 2 only

(a) 1 and 2 only (b) 2 and 3 only

24
(c) 1, 3 and 4 only (b) 2 and 4 only
(d) 1, 2, 3 and 4 (c) 3 and 4 only
(d) All of above
Q18. “While travelling through the desert of
Australia’s Outbacks, you come across an
Q21. Consider the following statements with
isolated, flat-topped hill or mountain with
reference to the moraines:
steep sides that is smaller in area than a
plateau.” 1. MorainesWhich composed of glacial till is
stratified and sorted debris.
The landform you encountered is called:
2. Lateral moraine forms along the sides of a
(a) Graben
glacier.
(b) Gloup
3. Ground moraine Composed mainly of clay
(c) Mesas and sand, it is the most widespread deposit of
(d) Fjord continental glaciers.
4. Medial moraines are formedWhen two
glaciers meet.
Q19. Consider the following statements with
reference to Drumlines: Which of the statements given above is/are
incorrect?
1. Drumlines are smooth oval shaped.
(a) 1 only
2. The long axes of drumlins are perpendicular
to the direction of ice movement. (b) 2 and 3 only

3. Drumlines give an Indication of direction of (c) 1 and 4 only


glacier movement. (d) All of the above
Which of the statements given above is/are
incorrect?
Q22. With reference to glacial erosional landform
(a) 1 only Cirque, which of the following statements
(b) 2 only is/are incorrect?

(c) 1 and 2 only 1. They are deep, long and wide troughs or
basins with very steep concaves to vertically
(d) 2 and 3 only
drop high walls at its head as well as sides.
2. A cirque is basically a V-shaped depression
Q20. Which of the following is/are Erosional formed by the erosional activity of a glacier.
Landforms in the Karst-Topography?
3. As glaciers can only originate above the
1. Pools snowline, studying the location of present-
2. Sinkholes day cirques provides information on past
glaciation patterns and on climate change.
3. Lapies
Select the correct answer using the codes given
4. Caves
below.
Select the correct answer using the codes given
(a) 1 only
below:
(b) 2 only
(a) 1 only

25
(c) 1 and 3 only Q25. Consider the following statements with
(d) 2 and 3 only reference to marine depositional landforms:
1. Beaches are permanent in nature.

Q23. Consider the following statements with 2. Off-shore bars are commonly formed across
reference to low sedimentary coasts: the mouth of a river.

1. Low sedimentary coasts are comparatively 3. The ridge of sand and shingle formed in the
shorter. sea is called an off-shore bar.

2. Bars, spits, and lagoons are features of low Which of the statements given above is/are
sedimentary coasts. incorrect?

3. Low sedimentary rocks are produced by (a) 1 only


upliftment of land. (b) 3 only
Which of the statements given above is/are correct? (c) 1 and 3 only
(a) 1 only (d) 1 and 2 only
(b) 1 and 2 only
(c) 2 and 3 only Q26. Which of the following statements is/are
(d) 1 and 3 only correct regarding landscapes developing in
running water?
1. Streams at the youth stage have large
Q24. With reference to marine erosional landforms,
floodplains.
which of the following statements is/are
correct? 2. In the mature stage, valleys look V shaped
and become shallow with waterfalls and
1. Rock debris, winds and tides are factors of
rapids.
marine erosion.
3. Natural levees, ox- bow lakes are features of
2. Differential erosion by sea waves through
old age streams.
rock with varying resistance across its
structure produces hanging valleys. Select the correct answer using the codes given

3. Notched cliff suffers maximum intensive below:

erosion by waves. (a) 1 only

Select the correct answer using the codes given (b) 3 only
below: (c) 1 and 2 only
(a) 2 only (d) 2 and 3 only
(b) 3 only
(c) 1 and 2 only Q27. Which of the following are the loop-like
(d) 1 and 3 only channel patterns that develop in the mature
stage of rivers?
(a) Horns

26
(b) Meanders 1. They are deep, long and wide troughs or
basins with very steep concaves to vertically
(c) Delta
drop high walls at its head as well as sides.
(d) Cirque 2. A cirque is basically a V-shaped depression
formed by the erosional activity of a glacier.

Q28. Consider the following statements with 3. As glaciers can only originate above the
reference to the Alluvial Fans: snowline, studying the location of present-
day cirques provides information on past
1. Alluvial fans are formedWhen streams glaciation patterns and on climate change.
flowing to higher levels break in the plains of
low gradients. Select the correct answer using the codes given
below.
2. Very coarse load is carried by streams
flowing over mountain slopes. (a) 1 only

3. Alluvial fans in dry areas show normally low (b) 2 only


conesWhile in humid areas they appear as (c) 1 and 3 only
steep slopes. (d) 2 and 3 only
Which of the statements given above is/are
incorrect?
Q31. Which ofthe following landforms are formed
(a) 3 only by glacial erosion?
(b) 1 and 2 only 1. Cirque
(c) 1 and 3 only 2. Arete
(d) 2 and 3 only 3. Pyramidal Peaks
4. Tarn
Q29. Consider the following statements with Select the correct answer using the codes given
reference to Meanders: below.
1. Meanders are loop-like patterns that develop (a) 1 only
over flood and delta plains.
(b) 1 and 3
2. There is active deposition along the convex
bank and erosion along the concave bank. (c) 2 and 4

3. Cut-off loops at inflection points are called (d) All of the above
ox bow lakes.
Which of the statements given above is/are correct? Q32. Consider the following statements with
(a) 1 only reference to wind in the evolution of
landforms:
(b) 1 and 3 only
1. Eddies,Whirlwinds, updrafts and downdraft
(c) 2 and 3 only are its features.
(d) All of above 2. Lifting and removal of dust and smaller
particles from the surface of rocks is
Q30. With reference to glacial erosional landform abrasion.
Cirque, which of the following statements 3. Torrential rain helps in removing weathered
is/are incorrect? debris.
Which of the statements given above are correct?

27
(a) 1 and 2 only (a) Winds
(b) 2 and 3 only (b) Waves
(c) 1 and 3 only (c) Rivers
(d) All of the above (d) Glaciers

Q33. Consider the following statements with


Q36. Which of the following are depositional
reference to pediments:
landforms of river terraces?
1. Pediments are concave in shape.
1. Alluvial fans
2. Pediments are limited to arid regions.
2. Deltas
3. Pediments are formed at the end of major
3. Cirque
tectonic cycles during the transition from the
orogenic stage of development of the earth’s 4. Eskers
crust to the platform stage. 5. Playas
Which of the statements given above are incorrect? Select the correct answer using the codes given
(a) 1 and 2 only below.

(b) 2 and 3 only (a) 1 and 2 only

(c) 1 and 3 only (b) 1, 3 and 4 only

(d) All of the above (c) 2, 4 and 5 only


(d) All of above

Q34. Consider the following statements with


reference to the depositional landforms in the Answer Key
desert region: 1. Ans: B 19. Ans: B
2. Ans: C 20. Ans: D
1. Transverse dunes are aligned parallel to the
3. Ans: D 21. Ans: A
wind direction. 4. Ans: C 22. Ans: B
2. Parabolic dunes are comparatively much 5. Ans: C 23. Ans: C
wider than barchans. 6. Ans: B 24. Ans: D
7. Ans: B 25. Ans: A
3. Longitudinal dunes are formedWhen the 8. Ans: A 26. Ans: B
supply of sand is rich. 9. Ans: B 27. Ans: B
10. Ans: D 28. Ans: A
Which of the statements given above are incorrect? 11. Ans: D 29. Ans: B
(a) 1 and 2 only 12. Ans: D 30. Ans: B
13. Ans: B 31. Ans: D
(b) 2 and 3 only 14. Ans: D 32. Ans: C
(c) 1 and 3 only 15. Ans: C 33. Ans: B
16. Ans: B 34. Ans: D
(d) All of the above 17. Ans: D 35. Ans: C
18. Ans: C 36. Ans: A

Q35. Natural levees and point bars are a


depositional landform ofWhich of the
following geomorphic regimes?

28
Geomorphic processes
Q1. With reference to Magma, consider the Which of the statements given above is/are correct?
following statements:
(a) 1 only
1. The viscosity of the magma that erupts from
(b) 1 and 3 only
a volcano affects the shape of the volcano.
(c) 3 only
2. Basaltic magma is high in iron and
magnesium. (d) 2 and 3 only

3. Lava that is extruded onto the surface during


a volcanic eruption is called Magma. Q4. With reference to the Weathering Processes,
consider the following statements:
Which of the statements given above are correct?
1. Weathering involves the transportation of
(a) 2 and 3 only
eroded particles from one place to another.
(b) 1 and 3 only
2. Temperature change is responsible for the
(c) 1 and 2 only weathering of the rocks.
(d) All of the above 3. The process of weathering is only Physical in
nature.
Q2. Consider the following statements about the Which of the statements given above is/are correct?
Geomorphic Processes:
(a) 1 and 2 only
1. Exogenic forces are considered as land
(b) 2 only
wearing forces.
2. Diastrophic movements are often associated (c) 2 and 3 only
with mountain building. (d) 1 only
3. Endogenic forces originate inside the earth.
Which of the statements given above is/are correct? Q5. Consider the following pairs:
(a) 1 and 2 only Weathering Characteristics
Processes
(b) 3 only 1. Carbonation A process that causes rust
(c) 2 and 3 only 2. Thawing A type of chemical
weathering
(d) All of the above
3. Oxidation Important in formation of
caves
Q3. Consider the following statements about
Magma: Which of the pairs given above is/are correctly
matched?
1. Lava with low silica content is thick and
(a) 1 and 2 only
viscous.
(b) 2 and 3 only
2. As the magma cools, its viscosity decreases.
(c) 1 only
3. The violent eruptions of the volcano are due
(d) None of the above
to high viscosity of the magmas.

29
Q6. Consider the following statements about 1. Rhyolitic Explosive eruptions with
Weathering and Erosion: high silica content
1. Mass wasting is an endogenic process.
2. Andesitic Shield volcanoes formed
2. Gravity is an agent of erosion. by Basaltic Magma
3. Humans are an agent of Weathering.
3. Basaltic Low silica content
Which of the statements given above are correct?
(a) 1 and 2 only
Which of the pairs given above is/are correctly
(b) 2 and 3 only matched?
(c) 1 and 3 only (a) 3 only
(d) All of the above. (b) 1 and 2 only
(c) 2 and 3 only
Q7. Consider the following statements:
(d) 1 and 3 only
1. Weathering of rocks helps in the enrichment
of certain ores.
Q10. Consider the following statements:
2. Chemical weathering causes physical
damage to rock. 1. Karst topography is produced due to the
chemical weathering.
Which of the statements given above is/are
incorrect? 2. The action of endogenic forces is uniform
throughout the earth.
(a) 1 only
3. Chemical weathering is the first stage in the
(b) 2 only production of soils.
(c) Both 1 and 2 Which of the statements given above is/are correct?
(d) Neither 1 nor 2 (a) 1 and 2 only
(b) 2 and 3 only
Q8. Consider the following statements:
(c) 1 and 3 only
1. Solifluction happens most in mountainous
(d) 3 only
places where a long dry season is followed by
heavy rains.
2. Rock slides happen in mountainous regions Q11. Consider the following pairs:
where mining is done.
Type of Features
3. Undercutting of steep slopes can trigger Mass
Erosion
Movements.
Which of the statements given above is/are correct? 1. Sheet Erosion through various
(a) 1 only erosion narrow and zig-zag channels

(b) 1 and 3 only 2. Gully Deep channels are associated


(c) 2 and 3 only Erosion features
(d) All of the above 3. Rill A complete layer is carried
Erosion along with water in a larger
Q9. Consider the following pairs: area
Types of Magma Characteristics Which of the pairs given above is/are correctly
matched?

30
(a) 1 and 2 only Q14. Consider the following statements with respect
(b) 2 only to Soil formation and weathering:
1. Topography is an essential active factor in
(c) 1 and 3 only
the Soil formation process.
(d) All of the above.
2. Plants play an active role in soil formation
through Chemical Weathering.
Q12. With respect to Weathering, consider the
3. Erosion plays an important role in Soil
following statements:
redistribution
1. Salt Wedging occurs when salts crysallize
Which of the statements given above is/are correct?
out of solution as water condenses.
(a) 1 and 2 only
2. Hydration is an irreversible chemical
weathering of rocks. (b) 2 and 3 only
(c) 3 only
Which of the statements given above is/are
incorrect? (d) All of the above

(a) 1 only
(b) 2 only Q15. Consider the following pairs:

(c) Both 1 and 2 Agent of Erosion Associated Landform

(d) Neither 1 nor 2 1. Wind Yardang


2. River U shaped valleys

Q13. In the context of Endogenic Forces, consider 3. Glaciers Alluvial Delta


the following statements: Which of the pairs given above is/are correctly
1. Epeirogeny is a mountain building process matched?
whereas Orogeny is continental building (a) 1 only
process.
(b) 3 only
2. Diastrophic movements are gradual and very (c) 1 and 3 only
slow in nature.
(d) All of the above.
3. Endogenic forces are usually driven by
gravity or atmospheric forces.
Q16. Which of the following statements is/are
Which of the statements given above is/are
correct?
incorrect?
1. Diastrophism and volcanism are exogenic
(a) 1 and 2 only
geomorphic processes.
(b) 2 only
2. Diastrophism refers to deformation of the
(c) 1 and 3 only Earth's crust due to deforming movements
(d) 2 and 3 only such as folding, faulting, warping and
fracturing.
Select the correct answer using the code given
below:
(a) 1 only

31
(b) 2 only Q19. Which of the following statements is/are
(c) Both 1 and 2 correct about landforms?

(d) None of the above 1. The stalagmite and stalactites eventually fuse
to give rise to columns and pillars of different
diameters.
Q17. Which of the following factors control the
2. Cirques are the most common of landforms
formation of soils?
in glaciated mountains.
1. Parent material
Select the correct answer using the code given
2. Topography
below:
3. Climate
(a) 1 only
4. Biological activity
(b) 2 only
5. Time
(c) Both 1 and 2
Select the correct answer using the code given
(d) None of the above
below:
(a) 1, 3, 4 and 5 only
Q20. Which of the following statements is/are
(b) 2 and 4 only
incorrect?
(c) 1, 2 and 5 only
1. Serrated Ridges form through headward
(d) All of the above erosion of the cirque walls.
2. Erosional forms dominate the west coast and
Q18. Which of the following statements is/are depositional forms dominate the east coast.
correct? Select the correct answer using the code given
1. Movement of water-saturated clayey or silty below:
earth materials down low-angle terraces or (a) 1 only
hillsides is known as earthflow.
(b) 2 only
2. Slump is the slipping of one or several units
(c) Both 1 and 2
of rock debris with a forward rotation with
respect to the slope overWhich the movement (d) None of the above
takes place.
Select the correct answer using the code given Q21. Which of the following statements is/are
below: incorrect?
1. All the endogenic geomorphic processes are
(a) 1 only
covered under a general term, denudation.
(b) 2 only 2. Exfoliation can occur due to expansion and
(c) Both 1 and 2 contraction induced by temperature changes.

(d) None of the above Select the correct answer using the code given
below:
(a) 1 only
(b) 2 only

32
(c) Both 1 and 2 Q24. Consider the following statements:
(d) None of the above 1. Debris avalanche is similar to snow
avalanche and it can be much faster than the
mudflow.
Q22. Which of the following statements is/are
correct about depositional landforms? 2. Mudflows occur frequently on the slopes of
1. Drumlins are smooth oval shaped ridge-like erupting or recently erupted volcanoes.
features composed mainly of glacial till with Which of the statements given above is/are
some masses of gravel and sand. incorrect?
2. Deposition and vertical erosion of banks are (a) 1 only
essential for the formation of braided
(b) 3 only
patterns.
(c) Both 1 and 2
Select the correct answer using the code given
(d) Neither 1 nor 2
below:
(a) 1 only
(b) 2 only Answer Key

(c) Both 1 and 2 1. Ans: C 13. Ans: C


2. Ans: D 14. Ans: B
(d) None of the above 3. Ans: C 15. Ans: A
4. Ans: B 16. Ans: B
5. Ans: D 17. Ans: D
Q23. Which of the following statements are 6. Ans: B 18. Ans: A
correct? 7. Ans: B 19. Ans: C
1. Shield Volcanoes are characterized by 8. Ans: C 20. Ans: A
outbreaks of cooler and more viscous lavas 9. Ans: D 21. Ans: A
than basalt. 10. Ans: C 22. Ans: A
11. Ans: B 23. Ans: B
2. When calderas erupt, they tend to collapse on 12. Ans: C 24. Ans: D
themselves rather than construct any
structure.
3. Flood Basalt Province volcanoes discharge
highly fluid lava that flows for long
distances.
Select the correct answer using the code given
below:
(a) 1 and 2 only
(b) 2 and 3 only
(c) 1, 2 and 3 only
(d) None of the above

33
Earthquake, volcano
Q1. Consider the following statements with Q4. With reference to earthquake consider the
reference to interior of the earth: following statements:
1. The core of Earth experiences only 1. Earthquake is diastrophic event.
compressional seismic waves. 2. Of Surface Waves, Rayleigh Waves is the
2. The shadow zone of p-waves is bigger than most Destructive and it produces entirely
the s-wave horizontal motion.
Which of the above statements is/are correct? Which of the following statement is/are correct?
(a) 1 only (a) 1 only
(b) 2 only (b) 2 only
(c) Both 1 & 2 (c) Both 1 & 2
(d) Neither 1 nor 2 (d) Neither 1 nor 2

Q2. With reference to Seismic waves, consider the Q5. Which of the following statement is/are
following statements: correct?
1. Deeper the focus of an earthquake, the 1. Earthquake swarm is a series of high
stronger the surface wave is. magnitude earthquakes occurring in a
2. The absence of p waves between 104 degree localised region over a period of time ranging
and 140 degrees led to the deduction that from days to weeks to even month.
earth has a liquid outer Core.
2. Earthquake swarm can serve as marker for
Which of the above statements is/are correct?
location of flowing magma.
(a) 1 only
3. Surface waves which are responsible for the
(b) 2 only
most destruction is high frequency, short
(c) Both 1 and 2 wavelength transverse waves.
(d) Neither 1 nor 2
Select the correct answer from the code given
below:
Q3. Which of the following statement regarding
type of Volcanoes is incorrect? (a) 1 only
(a) Stratovolcano have usually conical vent and (b) 2 only
associated with explosive eruption.
(c) 1 and 3 only
(b) Shield type Volcano have usually linear vent
and associated with quiet or sometimes no (d) All of the above
explosive eruption.
(c) Shield type Volcano becomes explosive if Q6. With reference to the measuring of
water gets into the vent. earthquakes, consider the following
(d) Stratovolcano are found at constructive plate statements:
margins while Shield type volcanoes are 1. Richter scale or the Magnitude Scale
found at destructive plate margins. measures the Magnitude of the damage
caused by the earthquake.
2. Mercalli Scale or the Intensity Scale
measures the Intensity of the energy released
during the Earthquake.

34
3. Richter scale has a variation of 1 – 10 while 2. Both Geysers and Fumaroles are
Mercalli scale has of 1-12. characterized by release of steam.

Which of the following statement is/are correct? 3. Hot springs Feature spring heated by
geothermally heated groundwaterWhile
(a) 1 and 2 only Geysers are spring featured by turbulent
(b) 3 only ejection of water.

(c) All of the above 4. Geysers show distinct colour due to presence
of cyanobacteria while Hot springs are
(d) None of the above colourful due to presence of silica deposit.
Which of the following statement are correct?
Q7. Which of the following statements are Correct (a) 1, 2 and 3 only
regarding Earthquake Prone Zones in India:
(b) 2, 3 and 4 only
(a) Ministry of Earth Science has divided India
(c) 1, 3 and 4 only
into four Seismic Zone.
(d) 1, 2 and 4 only
(b) Zone V is the most seismically active zone
while Zone II is the least seismically active
Q10. Consider the following statements:
zone.
1. Exfoliation domes are large and pointed
(c) Area covered under Zone III is the most formed as a result of weathering.
while Zone V covers the least share of total
2. Drastic diurnal temperature change aids the
seismically active area.
process of exfoliation making dry climate
(d) Zones are divided on the basis of Modified and high elevation most effective location for
Richter Scale which measures the magnitude exfoliation.
of earthquake. 3. Shattering produces very round pieces of
rock with blunt edges.

Q8. Consider the following statement about Which of the following statements is/are correct?
Tsunamis: (a) 1 only
1. Tsunami waves travel at different speeds in (b) 2 and 3 only
water - faster in deep water & slower in (c) 2 only
shallow water.
(d) 1 and 3 only
2. Amplitude of tsunami waves is more in deep
water than in shallow water.
Q11. With reference to Earthquake and Volcanic
Which of the following statement is/are correct? Eruption, which of the following statement is
incorrect?
(a) 1 only
(a) The remote Barren Island of India is the only
(b) 2 only confermed active volcano in the India
(c) Both 1 and 2 Subcontinent

(d) Neither 1 nor 2 (b) Mediterranean region is second most active


volcanic activity zone next to Circum Pacific
belt.
Q9. Consider the following statements: (c) Ring of Fire or Circum Pacific belt is a
1. Hot springs, Geysers & Fumaroles display subduction zone in entirety along its stretch.
release of underground energy. (d) Recent research shows cooling off of Pacific
plate in ring of fire.

35
Q12. With reference to denudational process, Q14. With Reference to Karst landform and Marine
consider the following statements: landform, consider the following statements:
1. Weathering aids both in Soil formation and 1. Karst landform is limited to area where
Enrichment
comparatively soluble rocks exist
2. Weathering is a prerequisite for mass
movement 2. Disappearance of streams in limestone
regions are related to sinkhole, swallow hole
3. Mass movement doesn't involve any
geomorphic agents and blind valley

Which of the following statements is/are correct? 3. Submerged coasts are dominated by
(a) 1 only depositional feature while Emerged coast are

(b) 2 and 3 only dominated by Erosional features

(c) 1 and 3 only 4. Tombolo is a depositional marine landform


(d) None of Above. in which an Island is attached to mainland by
a narrow piece of land

Q13. With reference to Riverine landform, consider Which of the following statements are correct?
the following statements: (a) 1, 2 and 4 only
1. Gorges are almost equal in width at both top (b) 2, 3 and 4 only
and bottom unlike Canyons which is wider at
(c) 1, 2 and 3 only
top than its bottom.
(d) 1, 2, 3 and 4
2. Retreat of waterfall can result into formation
of a gorge with steep sides.
Q15. Consider the following statements:
3. Incised meanders can be both symmetric or
asymmetric. Entrenched meanders are 1. Terrestrial planets are made up of rock and
symmetrical while Ingrown meanders are metals, and have relatively high densities.
asymmetrical. 2. Jovian planets have a thick atmosphere,
4. Alluvial Fans doesn't have clear stratification mostly of helium and hydrogen.
while Deltas have clear Stratification. 3. Jovian planets were formed after terrestrial
5. Point bars or meander bars are sediments planets so they are big in size.
deposited in semicircular fashion by flowing Which of the statements given above is/are correct?
rivers.
(a) 1 only
Which of the above statements are correct?
(b) 2 and 3 only
(a) 1, 3, and 5 only
(c) 1, 2 and 3
(b) 2, 4, and 5 only
(d) 1 and 2 only
(c) 1, 2, 3, and 4 only
(d) 3, 4, and 5 only Q16. With the reference to the structure of the
Earth, consider the following statements:
1. The crust is brittle in nature and thickness is
same at any place of the earth.

36
2. The Mantle has a density higher than the
crust and the lower mantle is liquid in state.
3. The core is made up of very heavy material
mostly constituted by nickel and iron.
Which of the statements given above are incorrect?
(a) 1 and 2 only

(b) 2 and 3 only

(c) 1 and 3 only

(d) 1, 2 and 3

Q17. In the context of volcanic eruptions, which of


the following statements is/are correct?
1. Shield Volcanoes are the largest of all the
volcanoes and mostly made of basalt.
2. Calder volcanoes are the most explosive
volcanoes.
3. Lava of composite volcanoes has large
quantities of pyroclastic material and ashes.
Select the correct answer using the code given
below:
(a) 2 only

(b) 2 and 3 only

(c) 1, 2 and 3

(d) 1 and 2 only

Answer Key
1. Ans: A 10. Ans: C
2. Ans: D 11. Ans: C
3. Ans: D 12. Ans: C
4. Ans: B 13. Ans: C
5. Ans: B 14. Ans: A
6. Ans: D 15. Ans: D
7. Ans: B 16. Ans: A
8. Ans: A 17. Ans: C
9. Ans: A

37
Relief of ocean
Q1. With reference to the continental shelf (d) A-3, B-5, C-4, D-2, E-1
consider the following statements: Q3. With reference to ocean deposits consider the
1. These are part of the oceanic crust. following statements:
2. It is broad where the mountains are away 1. Sediment deposits are thin or absent in the
from the coast and narrow where the mid oceanic ridges and are thickest near
mountains are nearer to the coast. thecontinents.
3. Continental shelves comprise half of the total 2. Terrigenous sediments consist of silts and
ocean bottom. clays whereas pelagic sediments are remains
4. The region is characterized by unstable water of marine organisms.
conditions. 3. Glacial marine sediments are primarily found
Which of the above statements are correct? in higher latitudes within the continental
shelf.
(a) 1, 2 and 3 only
Which of the statements given above is/are correct?
(b) 1, 3 and 4 only
(a) 1 and 2 only
(c) 2, 3 and 4 only
(b) 3 only
(d) 2 and 4 only
(c) 2 and 3 only
(d) 1, 2 and 3
Q2. Match the following pairs:
(List-I) (List-II)
Ocean relief Definition Q4. Which of the following statements regarding
A. Continental 1. Deep valleys found the temperature of the ocean are correct?
slope cutting across the
continental shelf 1. With increasing salinity, the temperature
B. Ocean 2. Zone of steep slope absorbing capacity is increased.
Deep extending from 2. Ocean water freezes at lower temperature
continental shelf. than freshwater.
C. Seamounts 3. Submerged, 3. The vertical distribution of temperature in the
isolated, flat- ocean shows a decrease in temperature with
topped mountains.
depth everywhere around the world.
D. Guyots 4. Submerged,
isolated, pinnacled 4. Mean temperature of the Indian ocean is
mountains. higher than the Pacific Ocean.
E. Submarine 5. Steep sided narrow
Select the correct answer from the code given
canyons basins, which are
the deepest part of below:
the ocean. (a) 1, 2 and 4 only
Select the correct answer from the codes given (b) 1 and 2 only
below. (c) 2 only
(a) A-2, B-3, C-1, D-5, E-4 (d) 2 and 3 only
(b) A-3, B-2, C-1, D-4, E-5
(c) A-2, B-5, C-4, D-3, E-1

38
Q7. Consider the following pairs:
Q5. In the context of salinity of the ocean, match
List-I and List-II: (List-II) (List-II)
List-I List-II Oceanic Trenches Region
A. Mariana trench Atlantic Ocean.
A. Halocline 1. Transition zone that
zone separates maximum B. Puerto Rico Pacific Ocean.
and minimum trench
salinities. C. Eurasia basin Indian Ocean.
B. Mixed zone 2. Upper layer of the D. Java trench Arctic Ocean.
ocean with relatively Which of the pairs given above are correctly
constant matched?
temperature (a) 1, 2 and 3 only
and salinity. (b) 1, 3 and 4 only
C. Thermocline 3. Boundary region of (c) 2, 3 and 4 only
zone surface water and (d) None of the above.
deep water marked
by rapid
temperature fall. Q8. The term andesite line relates to:
D. Pycnocline 4. Extreme fall in (a) The boundary line between continental slope
zone temperature and and the continental shelf.
increase in density (b) The ocean ward margin of continental
with depth. shelfWhere rapid increase in slope angle
E. Deep zone 5. Rapid increase of occurs.
water density with (c) The undulating plain lying beyond the
continental rise, covering two-third of the
depth due to
ocean floor.
temperature / (d) The areaWhere polymetallic nodules of
salinity change. copper, magnesium and cobalt are found on
Select the correct answer fromthe codes given seafloor.
below:
(a) A-1, B-2, C-3, D-4, E-5 Q9. Consider the following statements with respect
(b) A-1, B-2, C-3, D-5, E-4 to Oceans:
(c) A-2, B-1, C-3, D-4, E-5
(d) A-1, B-3, C-2, D-4, E-5 1. Among oceans the Atlantic Ocean covers
maximum of the earth surface and Indian
ocean the least surface area.
Q6. Consider the following statements: 2. Pacific Ocean has the highest average depth
followed by the Indian ocean.
1. Both the diurnal and annual range of
temperature of the ocean shows a lot of 3. Sequence of total salt contents found in
variation. oceanic water in descending order is: Sodium
Chloride, Magnesium Chloride, Magnesium
2. Annual range of temperature is greater in the
sulphate, Calcium sulphate, Potassium
Northern hemisphere than that in the
sulphate.
Southern hemisphere.
4. Oceanic surface is heavier and denser than
Which of the statements given above is/are correct? the continental surface.
(a) 1 only Which of the statements given above are correct?
(b) 2 only
(a) 1, 2, and 4 only
(c) Both 1 and 2
(b) 1 and 3 only
(d) Neither 1 nor 2
(c) 2, 3 and 4 only
(d) All of the above.

39
(a) 1 only
(b) 2 only
Q10. With reference to thermohaline circulation (c) Both 1 and 2
which of the following statements is/are (d) Neither 1 nor 2
correct?
1. Driven by the differences in the density of sea Q13. With reference to the Coral reefs, consider the
water and also known as subsurface currents. following statements:
2. Ocean bottom relief greatly influences
1. Agricultural runoff harms the growth of coral
thermohaline circulation.
reefs.
3. It is the mechanism behind the global
2. Second largest barrier reef is present in
conveyor belt.
Central America.
4. Gulf stream current is an example of
thermohaline circulation. Which of the above statements is/are correct?

Select the answer using the codes given below: (a) 1 only
(b) 2 only
(a) 1, 2 and 4 only
(c) Both 1 and 2
(b) 2 and 4 only
(d) Neither 1 nor 2
(c) 3 only
(d) 1, 2, 3 and 4
Q14. If someone is moving shoreline to seaward and
reaching in the mid ocean, which of the
Q11. Which of the following phenomenon can be
following order of topography s/he follows?
attributed to ocean currents?
1. Continental slope
1. Inducing heavy snowfall.
2. Abyssal plain
2. Discouraging rainfall.
3. Continental shelf
3. Helps in breeding of marine organisms.
4. Continental rise
4. Hinders navigation.
5. Harnessing renewable source of energy. Select the correct answer using codes given below:

Select the correct answer using the codes given (a) 1-4-2-3
below: (b) 3-1-4-2
(c) 2-3-4-1
(a) 1, 2 and 5 only
(d) 3-4-1-2
(b) 1, 2, 3 and 4 only
(c) 2, 3 and 5 only
(d) All of the above Q15. Which of the following are the major features
of the Ocean Floor?
1. Canyons
Q12. Which of the following statements is/are 2. Ridges
correct regarding the temperature of the 3. Oceanic deep
oceans? 4. Continental rise
1. The reduction of temperature with latitude is Select the correct answer using codes given below:
constant. (a) 1, 3 and 4 only
2. The temperature of the oceans varies (b) 1, 2 and 3 only
vertically with increasing depth. (c) 3 and 4 only
(d) 2, 3 and 3 only
Select the answer using the codes given below:

40
Which of the following topography is described
Q16. Which of the following are the minor features above?
of the Ocean Floor? (a) Continental Rise
1. Deep sea plain (b) Ocean Deep
2. Guyot (c) Abyssal plain
3. Sea mount (d) Shoals.
4. Canyon
5. Trench Q20. Consider the following pairs:
Select the correct answer using codes given below: Relief Description
(a) 1, 2 and 5 only 1. Continental marginal area submerged
(b) 2, 3, 4 and 5 only Slope in the water
(c) 1, 3 and 5 only 2. Oceanic steep slope from
(d) 2, 3 and 4 only ridge continental shelf to
abyssal plain
Q17. With reference to Continental shelf, consider 3. Abyssal Hills underwater hills more
the following statements: than 1000 m high
1. It is the shallowest part of the Ocean. Which of the following pairs are correctly
2. Its width from one ocean to another remains matched?
the same. (a) 1 only
3. Depths of the continental shelf are (b) 1 and 2 only
continuous in every area. (c) 3 only
Which of the statements given above is/are correct? (d) 1 and 3 only
(a) 1 and 2 only
(b) 1 only Q21. Consider the following statements:
(c) 2 and 3 only
(d) 3 only 1. The deep sea plain has an optimum amount
of clay and silt.
2. Oceanic deeps are very sensitive to
Q18. Consider the following statements: earthquakes and volcanoes.
1. Guyot is a mountain that has a pointed 3. The largest number of deeps and trenches
summit. found in the Pacific Ocean.
2. Seamount is a tabletop or flat top mountain
Which of the statements given above is/are correct?
which is commonly found in the Pacific
Ocean. (a) 1 only
Which of the following statements is/are correct? (b) 2 and 3 only
(c) 1 and 2 only
(a) 1 only
(d) 1, 2 and 3
(b) 2 only
(c) Both 1 and 2
(d) Neither 1 nor 2
Q22. With the reference to minor relief features of
oceans, consider the following statements:
Q19. In the ocean this topography lies in the depth 1. Seamount is mountain and volcanic in
of 3000-6000 meters, covering 40% of the total origin.
area of the Ocean basins. It is almost
featureless and their slope gradient is less than 2. The Pacific Ocean has the highest guyots.
1%. 3. Hudson canyon is the best-known submarine
canyon in the world.

41
4. Atolls are low islands found in temperate 1. Cape Verde Island, Bermuda island is
oceans. volcanic in origin and St. Helena is coral in
Which of the statements given above are correct? origin.
2. The highest of trenches found in the Atlantic
(a) 1, 2 and 3 only
Ocean.
(b) 2 and 4 only
Which of the statements given above is/are
(c) 1 and 3 only incorrect?
(d) 1, 2, 3 and 4 (a) 1 only
(b) 2 only
(c) Both 1 and 2
Q23. Consider the following statements:
(d) Neither 1 nor 2
1. Continental shelves are the source of fossil
fuels.
2. Coast of Chile and the West coast of Sumatra Q26. With reference to submarine Canyon,
has a very wide range of continental shelves. Consider the following statements:
3. About one-fourth of the world's petroleum 1. Submarine Canyon is generally found
and natural gas is extracted from the transverse to the coasts and in front of the
continental shelves. mouth.
Which of the statements given above is/are correct? 2. The gradient of submarine Canyon is
(a) 1 and 3 only shallower than the Continental Canyons.
(b) 2 only 3. Canyons found in the Indian Ocean are
(c) 1 and 2 only believed to be formed during the Cenozoic
(d) 3 only and Neozoic era.
Which of the following statements given above
is/are correct?
Q24. Which of the following statements is/are
correct? (a) 1, 2 and 3
1. Shoals are formed by erosional activity and (b) 2 and 3 only
coasts are formed by depositional activity. (c) 1 and 2 only
2. Reefs are associated with organic depositions (d) 1 and 3 only
of corals.
3. Coast is the bank of the ocean that consists of
only unconsolidated material.
Answer Key
Select the correct answer using the code given
1. D 10. D 19. C
below.
2. C 11. D 20. C
(a) 1 and 2 only 3. D 12. B 21. D
(b) 2 only 4. B 13. C 22. A
(c) 2 and 3 only 5. B 14. B 23. A
(d) 1, 2 and 3 6. B 15. C 24. B
7. D 16. B 25. C
8. A 17. B 26. D
Q25. With reference to the bottom Relief features of 9. C 18. D
the Atlantic Ocean, Consider the following
statements:

42
Movements of ocean
Q1. Which of the following statements is/are (c) 1 and 3 only
correct about the oxygen solubility in ocean
(d) All of the above
water?
1. It increases with an increase in ocean
temperatures. Q4. Consider the following statements about the
2. Oxygen solubility decreases with an increase influence of Coriolis force on oceans:
in salinity. 1. The Coriolis force intervenes and causes the
Select the correct answer from the code given water to move to the left in the southern
below: hemisphere and to the right in the northern
hemisphere.
(a) 1 only
(b) 2 only 2. Sargasso Sea is the result of circular current
(c) Both 1 and 2 due to Coriolis force
(d) Neither 1 nor 2 Which of the following statements is/are correct?
(a) 1 only
Q2. Consider the following statements: (b) 2 only
1. Salinity increases the porosity of the soil and (c) Both 1 and 2
the water-holding capacity of the soil.
(d) Neither 1 nor 2
2. Estuaries have lower salinity than open
oceans.
Q5. Arrange the following ocean currents from
Which of the statements given above is/are
west to east:
incorrect?
1. Humboldt Current
(a) 1 only
2. Benguela Current
(b) 2 only
3. California current
(c) Both 1 and 2
4. Canary current
(d) None of the above
Select the correct answer using the code given
below.
Q3. Consider the following statements about (a) 1-2-3-4
mineral resources available in the Indian
Ocean region (b) 3-4-2-1

1. Manganese nodules in the Indian Ocean (c) 1-2-4-3


cover a large area, over 100 million sq.km. (d) 3-1-4-2
2. They are found along the central Indian ridge.
3. World Ocean Council is the authority that Q6. Consider the following statements regarding
allocates licenses for exploration in open seas ocean deposits:
Which of the above statements is/are incorrect? 1. Placer deposits are lighter minerals that have
been weathered and eroded.
(a) 1 and 2 only
2. Indian placer deposits contain the largest
(b) 2 and 3 only
monazite reserves in the world.

43
3. Kerala coast has the highest reserves of 2. Site for fossil fuel reserves
thorium in India. 3. Good source of Polymetallic
4. Thorium is extracted from monazite sands. 4. Increases the height of tides
Which of the above statements are correct? Select the correct answer using the code given
(a) 1 and 2 only below:
(b) 2 and 3 only (a) 1, 2 and 3 only
(c) 2, 3, and 4 only (b) 2, 3 and 4 only
(d) All of the above (c) 1, 2 and 4 only
(d) 1, 2, 3 and 4
Q7. Which of the following ocean currents enclose
the Sargasso Sea? Q10. These are deep valleys, some comparable to
1. North Atlantic Drift the Grand Canyon of the Colorado river. They
2. Canary Current are sometimes found cutting across the
continental shelves and slopes, often extending
3. Gulf Stream from the mouths of large rivers. Identify the
4. Labrador Current above oceanic feature?
Select the correct answer using the code given (a) Submarine Canyons
below: (b) Seamount
(a) 2 and 4 only (c) Trenches
(b) 1, 2 and 3 only (d) Guyots
(c) 1, 2, 3 and 4
(d) 2, 3 and 4 only Q11. Consider the following statements:
1. Cold Currents are usually found on the east
Q8. Consider the following statements: coast of continents at higher latitudes in the
1. The mixing of warm and cold currents Northern hemisphere.
favours the growth of planktons. 2. Warm Currents are usually found on the east
2. Cold ocean currents have an effect on desert coast of continents in the low and middle
formation coast regions of the subtropical latitudes.
continents Which of the statements given above is/are correct?
Which of the statement/s given above is/are (a) 1 only
incorrect? (b) 2 only
(a) 1 only (c) Both 1 and 2
(b) 2 only (d) None of the above
(c) Both 1 and 2
(d) Neither 1 nor 2 Q12. Consider the following statements:
1. Continental Shelf is the shallowest part of the
Q9. In the context of Ocean Reliefs, which of the ocean.
following is/are the advantages of Continental
shelves? 2. The width of the continental shelves remains
1. Provide Richest fishing Grounds the same in almost all the oceans.

44
Which of the statements given above is/are correct? 4. Ocean currents
(a) 1 only 5. Density
(b) 2 only 6. Coastline configuration
(c) Both 1 and 2 Select the correct answer using the codes given
(d) Neither 1 nor 2 below.
(a) 1, 2 and 3 only
(b) 1, 2, 3 and 4 only
Q13. Which statements among the following about
(c) 1, 2, 3, 4 and 5 only
minor ocean relief features is correct?
(d) All of the above
(a) Ridges are formed along a convergent
boundary
Q16. Consider the following statements regarding
(b) Trenches are formed along a divergent
horizontal distribution of temperature across
boundary
ocean waters:
(c) Canyons are a result of depositional
1. The highest temperature of ocean water is
landform.
recorded at the equator.
(d) None of the above
2. The ocean waters of the northern hemisphere
are warmer than those of the southern
Q14. Consider the following statements: hemisphere.

1. The outgoing tide along the coast and from Which of the statements given above is/are correct?
the bays and estuaries during the time (a) 1 only
between low tide and high tide is called a (b) 2 only
flood current. (c) Both 1 and 2
2. The incoming tide during the time between (d) Neither 1 nor 2
high tide and low tide is called an ebb current.

Which of the statements given above is/are correct? Q17. Consider the following statements regarding
(a) 1 only waves in ocean waters:
1. Wind is the main driving force behind waves.
(b) 2 only
2. Waves occur both in surface as well as deep
(c) Both 1 and 2
ocean waters.
(d) Neither 1 nor 2
3. The actual motion of the water beneath the
waves is circular.
Q15. Which of the following factors affect the Which of the statements given above are correct?
salinity of oceans? (a) 1 and 2 only
1. Evaporation and precipitation (b) 2 and 3 only
(c) 1 and 3 only
2. Temperature of the region
(d) 1, 2 and 3
3. Latitude

45
Q18. Consider the following statements regarding
tides in oceans:
1. Tides are horizontal movement of ocean
waters.
2. Each place on Earth experiences two high
tides and two low tides every day.
3. Tides are a result of the gravitational force of
the moon only.
Which of the statements given above is/are correct?
(a) 1 only
(b) 2 only
(c) 3 only
(d) None of the above

Answer Key
1. Ans: B 10. Ans: A
2. Ans: C 11. Ans: C
3. Ans: C 12. Ans: A
4. Ans: C 13. Ans: D
5. Ans: D 14. Ans: D
6. Ans: C 15. Ans: C
7. Ans: B 16. Ans: B
8. Ans: D 17. Ans: C
9. Ans: D 18. Ans: D

46
Composition of atmosphere
Q1. With reference to the structure of the 1. Carbon dioxide is one of the ‘variable gases’
atmosphere, consider the following statement. and 4th most abundant gas in the atmosphere.
1. The warmer the weather the thicker is the 2. Carbon dioxide in the atmosphere remains
troposphere. constant in all seasons.
2. There are marked seasonal temperature Which of the above statements is/are correct?
changes in the stratosphere.
(a) 1 only.
3. The Kennelly Heaviside layer present within
(b) 2 only.
the ionosphere reflects the medium radio
(c) Both 1 and 2
waves.
(d) Neither 1 nor 2
4. The International Space Station is situated in
the thermosphere.

Which of the statements given above is/are correct? Q4. With reference to the atmospheric pressure,
consider the following statements:
(a) 1, 2 and 3 only.
(b) 1 and 2 only. 1. Lower layers of the atmosphere have high

(c) 2 only. pressure.

(d) All of the above. 2. Atmospheric circulation accounts for the


majority of heat redistribution across earth’s
surface.
Q2. Consider the following statements: 3. The temperature gradients are much steeper in
1. Water vapour is the most variable of the winter than in summer in both hemispheres.
atmospheric gases. 4. Relative humidity of the atmosphere is
2. Majority of the water vapour lies at the bottom measured by a barometer.
12 km of the atmosphere. Which of the given statements above are correct?
3. Sun’s halo is produced by the refraction of
(a) 1, 2 and 4 only.
light in ice crystals in Cirrus clouds.
(b) 1 and 4 only.
Which of the above statements is/are correct? (c) 1, 2 and 3 only.
(a) 2 only (d) All of the above.
(b) 2 and 3 only
(c) 1 and 3 only
Q5. In the context of clouds, consider the following
(d) 1, 2 and 3 pairs:
(Name:) (Description:)
Q3. Consider the following statements with respect 1. Nimbostratus Dark, dull cloud,
to the presence of carbon dioxide in the clearly layered and
brings continuous
atmosphere:
rain, snow or sleet.

47
2. Altocumulus Wooly, bumpy A. Fohn 1. Alps (Europe)
clouds arranged in B. Chinook 2. Rockies (North
layers and indicate America)
fine weather. C. Berg 3. South Africa.
3. Cumulonimbus Known as thunder D. Khamsin 4. Egypt.
clouds and brings E. Harmattan 5. Guinea
convectional rain, Coast/Eastern part
lightning, thunder of Sahara.
4. Cirrostratus Uniformly grey, F. Samun 6. Iran.
thick and low cloud
Select the correct answer using the codes given
that reduces below:
visibility of aircraft.
(a) A-1, B-2, C-3, D-4, E-5, F-6
Which of the following pairs is/are incorrectly (b) A-2, B-1, C-3, D-4, E-5, F-6
matched? (c) A-1, B-2, C-5, D-4, E-4, F-6
(a) 1 and 2 only (d) A-2, B-1, C-5, D-4, E-6, F-3
(b) 3 only
(c) 4 only Q8. Match the following cyclones with their area of
occurrence:
(d) 2, 3 and 4 only
List-I(Cyclones) List-II(Regions)
A. Typhoons 1- China sea
Q6. Which of the following factors affect the
B. Hurricanes 2- USA
distribution of insolation?
C. Tornadoes 3- Caribbean Sea
1. Absorption. D. Baguio 4- Japan
2. Distance between the earth and the sun. E. Taifu 5- Philippines
3. Reflection. F. Willy Willies 6- Northern Australia

4. Angle of the sun’s rays. Select the correct answer from the codes given
below:
5. Length of the day.
(a) A-1, B-2, C-3, D-4, E-5, F-6
Select the correct answer from the codes given
(b) A-1, B-3, C-2, D-5, E-4, F-6
below:
(c) A-2, B-1, C-2, D-5, E-4, F-6
(a) 1, 2, 4 and 5 only (d) A-2, B-1, C-3, D-4, E-5, F-6
(b) 2, 3 and 4 only
(c) 1, 2, 3 and 4 only Q9. Consider the following statements regarding
(d) All of the above. the Jet streams:
1. The greater the horizontal temperature
difference, the stronger the Jet Streams.
Q7. Match the following local winds with the area
2. Determines local weather conditions.
of occurrence:
List-I List-II 3. It plays an important role in the onset and
(Local Winds Names) (Region) withdrawal of Indian monsoon.

48
4. It acts as a transporter of radioactive dust and Select the correct answer from the codes given
other pollutants. below:
Which of the statements given above is/are (a) 1 and 2 only
correct? (b) 2 only
(a) 1 and 3 only (c) 1 and 3 only
(b) 2 only (d) All of the above
(c) 2, 3 and 4 only
(d) All of the above Answer Key
1. Ans: D 7. Ans: A
2. Ans: D 8. Ans: B
Q10. With reference to the Life cycle of a Temperate
3. Ans: A 9. Ans: D
cyclone, match the following lists: 4. Ans: C 10. Ans: B
List-I (Stages) List-II (Characteristics) 5. Ans: C 11. Ans: D
A. First stage 1. Warm and cold air 6. Ans: D
masses penetrate
into each other.
B. Second 2. Convergence of two
stage air masses of
contrasting direction
and nature.
C. Third stage 3. Cyclone fully
minedeveloped.
D. Fourth 4. Warm sector is
stage narrowed.
E. Fifth stage 5. Warm sector
completely
disappears.
F. Sixth stage 6. Occlusion of cyclone
starts.
Select the correct answer from the codes given
below:
(a) A-1, B-2, C-3, D-4, E-5, F-6
(b) A-2, B-1, C-3, D-4, E-6, F-5
(c) A-1, B-2, C-4, D-3, E-5, F-6

Q11. Which of the following is/are the characteristics


of anticyclone?
1. Anticyclone moves clockwise in the Northern
Hemisphere and anticlockwise in the Southern
Hemisphere.
2. Weather becomes rainless, with clear skies.
3. Temperature of anticyclone is directly
proportional to the local weather.

49
Solar radiation and heat balance

Q1. Which of the following statements is/are 3. Higher latitudes make greater angle with the
correct? Earth’s surface.
1. The maximum insolation is received by Earth Select the correct answer using the codes given
during the month of July. below:
2. The amount of insolation received is related (a) 1 and 2 only
to the latitudinal position. (b) 2 and 3 only
Select the correct answer using the codes given (c) 1 and 3 only
below:
(d) 2 only
(a) 1 only
(b) 2 only
Q4. Consider the following statements:
(c) Both 1 and 2
1. The insolation received by the land part of the
(d) Neither 1 nor 2 Earth is called terrestrial radiation.
2. The absorption of terrestrial radiations by the
Q2. Consider the following factors which affect the atmospheric gases generates a greenhouse
intensity of isolation: effect.
1. Length of the day. Select the correct answer using the codes given
2. Rotation of Earth. below:

3. Atmospheric transparency (a) 1 only

4. Land configuration. (b) 2 only

5. Gravitational pull (c) Both 1 and 2

6. Axial tilt of earth (d) Neither 1 nor 2

7. Tidal waves
Q5. Which of the following statements is incorrect
Select the correct answer using codes given below:
regarding the heating of Earth?
(a) 1, 2, 3, 4, and 7 only
(a) The magnitude of heat reflected by the Earth
(b) 1, 2, 3, 5, and 6 only is determined by its albedo.
(c) 2, 3, 5, 6, and 7 only (b) The Earth returns the entire heat received to
(d) 1, 2, 3, 4, and 6 only the atmosphere to maintain constant
temperature.
(c) The maximum insolation is received by the
Q3. Which of the following statements is/are
Earth at the equator.
incorrect?
(d) There is a positive net radiation received near
1. Greater the area covered by the sun rays, the tropics.
more is the heat per unit area.
2. Slant rays suffer greater scattering and
diffusion compared to direct rays.

50
Q6. Consider the following statements: Q9. Which of the statements given below is correct
1. Earth’s atmosphere is heated directly by the regarding the forces acting on the air?
insolation of the sun. (a) Pressure gradient force is weak in places of
2. Normal lapse rate is responsible for lesser closer isobars.
temperature at higher altitudes. (b) Frictional force does not cause any effect in
3. Land portions of the Earth are subjected to the speed of the winds over landmasses.
greater temperature changes. (c) The Coriolis force deflects the wind to the
Which of the statements given above are correct? right in the southern hemisphere.

(a) 1 and 2 only (d) The magnitude of Coriolis force is directly


proportional to the latitudinal angle.
(b) 1 and 3 only
(c) 2 and 3 only
Q10. Consider the following events:
(d) 1, 2, and 3
1. Rotation of Earth.
2. Oscillating pressure belts.
Q7. Consider the following statements:
3. Distribution of land and sea.
1. The isotherms of the Southern Hemisphere
show greater deviation compared to the 4. Gravitational pull of the moon.
Northern hemisphere. 5. Sunspot cycle.
2. During winter in the Northern Hemisphere, Which of the above factors are responsible for the
the isotherms bend southwards along the distribution of the planetary winds?
continents. (a) 1, 2, and 4 only
Which of the statements given above is/are correct? (b) 2, 4, and 5 only
(a) 1 only (c) 1, 2, and 3 only
(b) 2 only (d) 2, 3, and 5 only
(c) Both 1 and 2
(d) Neither 1 nor 2 Q11. With reference to planetary winds, Consider
the following statements:
Q8. With reference to temperature inversion, 1. The Trade winds blow towards the Equator
consider the following conditions: while the westerlies blow towards the poles.
1. Air drainage 2. Inter-Tropical Convergence zone is created at
2. Distance from sea level the convergence of Westerlies and the Trade
3. Winter nights with clear sky winds.

Which of the above conditions are ideal for 3. The air flow around the equator is the
temperature inversion to take place? Easterlies.
(a) 1, 2, and 3 Select the correct answer using the codes given
(b) 2, and 3 only below:
(c) 2 only (a) 3 only
(d) 1 and 3 only (b) 1 and 2 only

51
(c) 1 and 3 only 3. Occluded fronts are formed when two cold
(d) 2 and 3 only air masses uplift a comparatively cooler air
mass.
Which of the statements given above is/are correct?
Q12. Consider the following pairs:
(a) 1 and 2 only
Pressure cells Location
(b) 2 only
1. Hadley Cell Subtropical high
(c) 1 and 3 only
2. Ferrel Cell Tropical low
(d) 1, 2 and 3
3. Polar Cell Polar High
Which of the above pairs are incorrectly matched?
Q15. Which among the following statements is
(a) 1 and 2 only correct regarding the Extra tropical cyclones?
(b) 2 and 3 only (a) They are thermally induced low pressure
(c) 1 and 3 only areas which move from west to east.
(b) They are initiated with the creation of an
(d) 1, 2, and 3
occluded front in the mid latitudes.
(c) They cover a small geographical area and
Q13. Consider the following statements: inflict maximum damage to it.
1. The atmospheric air circulation plays an (d) These cyclones have the capacity to originate
important role in the transfer of energy to on the land surface.
oceans.
2. There is a unidirectional transfer of energy
Q16. Consider the following statements regarding
from the atmospheric air to the ocean
tropical cyclones:
currents.
1. They are thermally induced low-pressure
Select the correct answer using the codes given systems travelling in the eastward direction.
below:
2. They can originate on the surface of a water
(a) 1 only body only.
(b) 2 only 3. They do not require any prior low pressure in
(c) Both 1 and 2 the region of their formation.

(d) Neither 1 nor 2 4. The tropical cyclones formed at the Equator


are most destructive in nature.
Which of the statements given above are incorrect?
Q14. Consider the following statements regarding
fronts: (a) 2, 3 and 4 only

1. Warm fronts are formed when a warm air (b) 1, 3, and 4 only
mass pushes and uplifts the colder one. (c) 2 and 3 only
2. Fronts are the areas of sharp changes in (d) 1, 2, 3 and 4
temperature and pressure.

52
Q17. Consider the following statements regarding
cyclones in India:
1. Frequency of cyclones is more in the Bay of
Bengal while destructivity is more of the
Answer Key
Arabian sea cyclones.
1. Ans: B 10. Ans: C
2. The lack of influx of freshwater on the 2. Ans: D 11. Ans: C
Arabian Sea is an important reason for its low 3. Ans: C 12. Ans: D
cyclonic activity. 4. Ans: B 13. Ans: A
5. Ans: C 14. Ans: B
3. Occurrence of Simultaneous cyclones is 6. Ans: C 15. Ans: D
responsible for the rise in the Accumulated 7. Ans: B 16. Ans: B
Cyclone Energy. 8. Ans: D 17. Ans: B
9. Ans: D 18. Ans: A
Which of the statements given above are correct?
(a) 1 and 2 only
(b) 2 and 3 only
(c) 1 and 3 only
(d) None of the Above

Q18. Consider the following statements with


reference to cyclone:
1. The anticyclonic winds in the Southern
hemisphere move in the anticlockwise
direction.
2. The pressure gradient force is greater
horizontally than vertically.
Which of the statements given above is/are correct?
(a) 1 only
(b) 2 only
(c) Both 1 and 2
(d) Neither 1 nor 2

53
Planetary winds
Q1. Which amongst the following statements is/are (d) A is false but R is true
incorrect?
Q3. With reference to Anticyclone, which of the
1. Trade winds are responsible for the China following statements is/are incorrect?
type of climate.
1. In the NCR region, Anticyclonic conditions
2. China's type of climate is found on eastern help to minimise the pollution in the winter
margins of continents in warm temperate season.
latitudes.
2. These are the common phenomena along the
3. Offshore trade winds are the main reason for equatorial belt.
formation of the Mid latitude desert.
3. No cloud or Cloudless weather is one of the
Select the answer using the codes given below: characteristics of anticyclonic conditions.
(a) 1 and 2 only Select the answer using the codes given below:
(b) 2 only (a) 1 only
(c) 1 and 3 only (b) 2 only
(d) 3 only (c) 1 and 2 only

(d) 1 and 3 only


Q2. The following question consist of two
statements, one labeled as the Assertion (Aand
Q4. Consider the following statements related to
the other as Reason (R). You are to examine
westerlies:
these two statements carefully and select the
answers to these items using the code given 1. Westerlies are characterized by decreasing

below: rainfall from east to west in the southern


hemisphere.
Assertion(A): Geostrophic wind moves
perpendicular to Isobar 2. Westerlies in the Northern hemisphere are
stronger and persistent than in the southern
Reason(R): The geostrophic flow is the
hemisphere.
theoretical wind that would result from an exact
balance between the coriolis force and the 3. Mediterranean climate is the result of North
pressure gradient force. south movement of westerlies.

Codes: Which of the statements given above is/are


incorrect?
(a) Both A and R are individually true and R is
the correct explanation of A (a) 1 and 2 only

(b) Both A and R are individually true but R is (b) 2 only


not the correct explanation of A (c) 2 and 3 only
(c) A is true but R is false (d) 1 and 3 only

54
Select the answer using the codes given below:
Q5. Consider the following statements related to (a) 1 and 2 only
Indian Ocean Dipole: (b) 3 only
1. El-Nino and the Negative IOD year have (c) 2 and 3 only
similar effects on Indian agriculture in terms
of production. (d) 1,2 and 3

2. Positive IOD results in more cyclones in


Arabian Sea than usual. Q8. Considering the following statements related
to Air Mass, which statements is/are incorrect?
Which of the statements given above is/are correct?
1. Anticyclonic areas are most ideal for regions
(a) 1 only
for development of air mass.
(b) 2 only
2. A pre-existing Low-level disturbance is
(c) 1 and 2 only essential for formation of air masses.
(d) Both 1 and 2 3. Air Mass has uniform temperature and
moisture conditions.
Q6. With reference to the Tropical cyclones and Select the answer using the codes given below:
thunderstorms, which of the following (a) 1 only
statements is/are correct?
(b) 1 and 3 only
1. Tropical cyclones and thunderstorm both
(c) 2 only
form because of intense heating of land
(d) 1 and 2 only
2. Heat thunderstorms are more common in the
belt of doldrum.
3. Updraft of air is stronger in tropical cyclone Q9. Consider the following statements related to
compared to thunderstorm formation of ITCZ, which of the following
is/are correct?
Select the answer using the codes given below:
1. ITCZ is the result of convergence of two
(a) 1 and 2 only
contrasting air masses.
(b) 2 only
2. ITCZ is formed over a subtropical high-
(c) 2 and 3 only pressure zone.
(d) 1,2 and 3 3. Mid latitude cyclone formation is one of the
characteristics of ITCZ.
Q7. With reference to Jet Stream, which of the 4. ITCZ is a broad rough of low pressureWhere
following statements is/are correct? air ascends.
1. Somali jet stream strengthens the south west Select the answer using the codes given below:
monsoon in India (a) 1, 2 and 3 only
2. During winters, an increase in thermal (b) 2, 3 and 4 only
contrast intensifies the jet stream. (c) 4 only
3. Heating of Tibetan plateau in summer (d) All of the above
intensify the Tropical easterly Jet Stream

55
Q10. With reference to El-Nino, which of the 2. Convergence of two different air mass is
following statements is/are incorrect? prerequisite condition for the formation of
1. It is characterised by further decrease in mid latitude cyclone.
temperature along the Peru and Ecuador 3. More the gentle slope of the front more the
coast. downpour and bad weather.
2. This brings drought in west pacific and more 4. Contrast in Temperature of two air mass is
rain to South American western coast more in summer session.
3. El-Nino year increase production of summer Select the answer using the codes given below:
crops in India (a) 1 and 4 only
Select the answer using the codes given below: (b) 2 only
(a) 1 and 2 only (c) 1 and 2 only
(b) 2 and 3 only (d) 3 and 4 only
(c) 1 and 3 only
(d) 1,2 and 3
Q13. Temperate cyclone does not form on equator
because:
Q11. With reference to westerlies, which of the (a) At the equator two air mass do not converge
following statement is correct?
(b) Uniform temperature condition of two air
1. Western margin of Britain get rainfall massesWhich converge at equator.
throughout the year.
(c) Coriolis force is maximum at the equator.
2. Between 30- 45 deg N/S latitude wet summer
(d) Doldrum at the equator inhabits the
and dry winter is a result of westerlies.
formation of cyclones.
3. Rainfall in the Laurentian climatic region is
the result of westerlies.
Q14. Which among the following is/are the direct
4. Westerlies are responsible for the rainfall in
implication of shifting of wind belts?
the North western part of India throughout
1. Monsoon climate in south East Asia region,
the year.
Indian sub continents and parts of Africa.
Select the answer using the codes given below:
2. Rainfall in western part of continents within
(a) 1, 2 and 3 only
zone 30 deg to 40 deg latitude in winter
(b) 2, 3 and 4 only
season.
(c) 1 only
3. In the summer solstice weakened polar
(d) All of the above
easterlies in northern hemisphere
Select the answer using the codes given below:
Q12. With reference to air mass and Fronts, which
(a) 1 and 2 only
of the following statements is/are correct?
(b) 2 only
1. Deformatory circulation is most favourable
for frontogenesis. (c) 1 and 3 only
(d) 1,2 and 3

56
3. They do not exercise any influence over the
Q15. Which among the following statement is weather conditions in India.
correct about the walker cell circulation? Select the answer using the codes given below:
(a) It is the result of temperature gradients from (a) 1 and 2 only
east west in Indian ocean.
(b) 2 and 3 only
(b) It is created by differential heating of land
(c) 2 and 3 only
and sea surface in Pacific Ocean.
(d) 1,2 and 3
(c) It is formed due to development of pressure
gradient from east to west in equatorial
Pacific Ocean. Q18. Which of the following phenomena is/are

(d) It is established as a result of thermohaline associated with the Indian monsoon during an
El-Nino year?
circulation around the globe.
1. Decrease in normal temperature of
Lakshadweep Sea.
Q16. Consider the following statements related to
2. Reduction in amount of moisture carried by
tropical and temperate cyclones is/are
trade winds coming from South America.
incorrect?
1. Temperate cyclone is relatively more 3. Strengthening of Inter Tropical Convergence

uniform and regular than the tropical cyclone Zone (ITCZ) across Northern plains.

2. Both cyclones can be associated with heavy Select the answer using the codes given below:
downpour, thunder and lightning (a) 1 only

3. Tropical and temperate cyclones originate (b) 2 only


only over the warm ocean in tropics and mid (c) 2 and 3 only
latitude respectively.
(d) 1 ,2 and 3
Select the answer using the codes given below:
(a) 1 and 2only
Q19. Which of the following phenomenon may
(b) 2 only cause rising of air?
(c) 2 and 3 only 1. Convection currents
(d) 3 only 2. Convergence of winds on land
3. Advection
Q17. Which of the following statements with 4. Eddies
reference to Jet Streams are correct?
Select the answer using the codes given below:
1. They are winds of upper air circulation that
(a) 1 and 4 only
blow from west to east.
(b) 2 ,3 and 4 only
2. Their pattern can be altered by the presence
of high mountains and highlands on the (c) 1,2 ,3 and 4
earth's surface. (d) 1 ,2 and 4 only

57
Q20. Consider the following statements related to (b) 1 and 3 only
Airmass: (c) 1, 2 and 3
1. Anticyclonic areas are most ideal for regions (d) 3 only
for development of air mass. Q23. Consider the following Pairs:
2. Source region for formation of air mass is Region Climate
land only. 1. The islands of East Indies- Tropical Monsoon
3. Homogeneous Earth surface is one of climate
essential conditions for formation of Air 2. Central Chile- Mediterranean Climate
mass. 3. Western equatorial Africa- Tropical Wet
Which of the statements given above is/are correct? Climate

(a) 1 only Which of the pairs given above is/are correctly


matched?
(b) 2 and 3 only
(a) 1 and 2 only
(c) 1 and 3 only
(b) 2 only
(d) 1,2 and 3 (c) 2 and 3 only
(d) 1, 2 and 3

Q21. Which of the following are the main areas


showing Marine west coast climate? Q24. This type of climate occurs over interior
1. West coast of North America Greenland. In summer the temperature is
2. Southern Chile below the freezing point and this area receives
3. South East of New Zealand very little precipitation. The snow and ice get
4. Northern Australia accumulated and the mounting pressure
causes the deformation of ice sheets.
Select the correct answer using the code given
below: Which of the following defines this type of climate?

(a) 1 and 3 only (a) Humid subtropical climate


(b) 1 only (b) Tundra Climate
(c) 1, 2 and 3 only (c) Ice cap Climate
(d) 1, 2, 3 and 4 (d) Marine West Coast Climate

Q22. With reference to Mediterranean Climate, Q25. With reference to Humid Subtropical Climate,
Consider the following statements: consider the following statements:
1. This type of Climate lies in the Eastern parts
1. This type of climate occurs along the west
of the continents in subtropical latitudes.
coast of continents in subtropical latitudes. 2. There is rain throughout the year in this
2. Major areas showing this type of climate are climatic region.
Central Chile and Northern Australia. 3. This type of climate occurs in Eastern China,
3. These areas come under tropical highs in Eastern coast of Australia.
summer and westerly winds in winter. Which of the statements given above is/are correct?
Which of the statements given above is/are correct? (a) 1 only
(b) 2 and 3 only
(a) 1 only

58
(c) 1 and 3 only (a) 1 and 3 only
(d) 1, 2 and 3 (b) 1 only
Q26. Consider the following statements regarding (c) 2 and 3 only
Prevailing winds: (d) 1, 2 and 3

1. These winds blow from a single direction


over a specific area of the Earth. Q29. With reference to Doldrums, Consider the
2. These winds generally blow North-South following statements:
rather than East-West. 1. The area around the Inter tropical
3. The blowing of such winds is because of Convergence Zone is known as Doldrums.
earth's rotationWhich is known as Coriolis 2. Prevailing winds in the doldrums are very
force. usually strong because of low pressure.
Which of the statements given above is/are correct? 3. The low-pressure doldrums are formed as the
(a) 1 and 2 only sun heats the equatorial region and causes air
(b) 1 and 3 only masses to rise and travel north and south.
(c) 2 and 3 only Which of the statements given above is/are correct?
(d) 1, 2 and 3 (a) 1 only
(b) 2 and 3 only
(c) 1 and 3 only
Q27. With reference to Westerlies, consider the
following statements: (d) 3 only

1. The tip of South America and Australia, as


well as the islands of New Zealand, are the Q30. Consider the following statements regarding
only large landmasses to penetrate the Jet Streams:
Roaring Forties.
2. Westerlies are strongest in the winter, when 1. They are formed near the boundaries of air
pressure over the pole is low, and weakest in masses with different temperatures and
summer, When the polar high creates humidity.
stronger polar easterlies.
2. The rotation of the Earth and its uneven
Which of the statements given above is/ are heating by the sun also contribute to the
correct?
formation of high-altitude jet streams.
(a) 1 only
3. Jet streams blow through a layer of the
(b) 2 only
(c) Both 1 and 2 atmosphere called the troposphere.
(d) Neither 1 nor 2 Which of the statements given above is/are correct?

(a) 1 only
Q28. Consider the following statements regarding (b) 1 and 2 only
Horse Latitudes:
(c) 1 and 3 only
1. They are narrow zones of warm, dry climates
(d) 1, 2 and 3
between westerlies and the trade winds.
2. Horse latitudes are about 40 and 45 degrees north Q31. With reference to Typhoons, Consider the
and south. following statements:
3. The Arid Kalahari of Africa are part of the horse
latitudes. 1. Typhoons are tropical storms and their

Which of the statements given above is/ are formation is identical to hurricanes.
correct?

59
2. Typhoons form as equatorial winds and blow Select the correct answer using the code given
westward before turning north and merging below:
with westerlies around the mid-latitudes. (a) 1 and 2 only
Which of the statements given above is/are correct? (b) 1, 2 and 4 only

(a) 1 only (c) 1 and 3 only

(b) 2 only (d) 1, 2 and 3 only

(c) Both 1 and 2


(d) None of the above Q35. Consider the following statements regarding
Anabatic Winds:
1. Anabatic wind is caused by thermal/heat
Q32. Consider the following statements regarding
processes.
Trade Winds: 2. Anabatic (upslopewinds occur over
1. Trade winds that form over ocean are warmer slopesWhich are heated by the sun.
and drier. Which of the statements given above is/are correct?
2. Trade winds are the powerful prevailing (a) 1 only
winds that blow from the east across the (b) 2 only
tropics. (c) Both 1 and 2
3. Most tropical storms, including hurricanes, (d) None of the above
cyclones, and typhoons, develop as trade
winds.
Q36. With reference to Katabatic winds, Consider
Which of the statements given above is/are correct? the following statements:

(a) 1 and 3 only 1. Katabatic winds are thoseWhich occur over


(b) 2 only slopesWhich are cooled.
2. Katabatic winds are nocturnal phenomena in
(c) 2 and 3 only
most parts of the world, especiallyWhen
(d) 1, 2 and 3
there is little cloud and due to lack of heating
by the sun.
3. Katabatic winds may lead to the formation of
Q33. Which of the following forms near the
frost, mist and fog in valleys.
boundaries of air masses with different
Which of the statements given above is/are correct?
temperatures and humidity?
(a) 1 and 3 only
(a) Trade winds
(b) 1 only
(b) Westerlies
(c) 1 and 2 only
(c) Jet streams (d) 1, 2 and 3
(d) Polar Easterlies
Q37. Which of the following are fed by polar
Q34. Which of the following are Hot winds? easterlies and winds from the high-pressure
horse latitudes?
1. Sirocco
2. The Santa Ana (a) Polar Easterlies
3. Foehn (b) Jet Streams
(c) Westerlies
4. Mistral

60
(d) Easterlies 3. Ans: C 23. Ans: C
4. Ans: A 24. Ans: C
5. Ans: C 25. Ans: D
Q38. Consider the following pairs regarding local 6. Ans: B 26. Ans: B
winds: 7. Ans: D 27. Ans: C
8. Ans: C 28. Ans: A
Local Name Region/Area 9. Ans: C 29. Ans: C
1. Harmattan Southern Africa 10. Ans: C 30. Ans: B
11. Ans: C 31. Ans: C
2. Khamsin Red Sea
12. Ans: C 32. Ans: C
3. Levanter Strait of Gibraltar 13. Ans: B 33. Ans: C
Which of the pairs given above is/are correctly 14. Ans: D 34. Ans: D
matched? 15. Ans: C 35. Ans: C
16. Ans: D 36. Ans: D
(a) 2 only 17. Ans: A 37. Ans: C
(b) 1 and 3 only 18. Ans: B 38. Ans: D
(c) 1 and 2 only 19. Ans: C 39. Ans: D
20. Ans: C
(d) 2 and 3 only

Q39. Consider the following statements regarding


Subtropical Steppe and Subtropical Desert
Climate:
1. Subtropical Steppe receives slightly more
rainfall than the desert.
2. Variability of rainfall is present in subtropical
steppe and deserts both.
3. In deserts, Rain occurs in short intense
thunderstorms and is ineffective in building
soil moisture.
Which of the statements given above is/are correct?
(a) 1 only
(b) 1 and 2 only
(c) 1 and 3 only
(d) 1, 2 and 3

Answer key
1. Ans: C 21. Ans: C
2. Ans: D 22. Ans: B

61
Wind circulation
Q1. Consider the following statements about Which of the above statements are correct?
Stratocumulus clouds:
(a) 1 and 2 only
1. They contribute to hothouse earth
(b) 1 and 3 only
phenomena.
(c) 2 and 3 only
2. These clouds are spread over 80% of
subtropical oceans. (d) 1, 2 and 3

Which of the above statement is/are correct?


(a) 1 only Q4. Consider the following statements about
Orographic rainfall:
(b) 2 only
1. This type of rainfall occurs due to the rise
(c) Both 1 and 2
of air because of the heating of mountain
(d) Neither 1 nor 2 slopes.
2. Most of the rainfall occurring in India is
Q2. Consider the following statements about Dew of this nature.
formation: 3. This type of rainfall leads to occurrence of
1. Dew forms when water in the atmosphere rain shadow areas.
reaches dew point temperature. Which of the above statements are correct?
2. Strong winds increase the possibility of dew (a) 1 and 2 only
formation.
(b) 2 and 3 only
3. Dew formation makes plants more resilient to
(c) 1 and 3 only
hot and dry conditions.
(d) 1, 2 and 3
Which of the above statements is/are correct?
(a) 1 only
Q5. Consider the following statements about the
(b) 1 and 2 only
type of fog and mechanism of their formation:
(c) 1 and 3 only
1. Advection fog is formed because of
(d) 2 and 3 only
cooling of ground and adjacent air.

2. Radiation fog is formed because of


Q3. Consider the following statements about transportation of warm, moist air over
Relative Humidity: cold surfaces.
1. It is a measure of how much moisture the air
3. Frontal fog occurs along the front
can hold.
separating cold and warm air masses.
2. Relative humidity depends on the actual
Which of the above statement is/are correct?
moisture content of the air, temperature and
barometric pressure. (a) 1 only
3. On average, the annual relative humidity of (b) 1 and 2 only
Chennai and Delhi is equal.

62
(c) 3 only Q8. Which of the following statements about

(d) 2 and 3 only Trade winds is correct?

Q6. Consider the following pairs of local winds and (a) They generally blow in the upper atmosphere

region in which they blow: at the altitude of 8 to 15 km.


(b) They blow from an equatorial high-pressure
Local wind Region
belt to sub-tropical low-pressure belt.
1. Chinook Switzerland (c) They are associated with heavy rainfall near
the equator.
2. Zonda Argentina (d) None of the above

3. Santa Ana California


Q9. Consider the following statements about
4. Berg Luxemburg Westerlies:
1. They blow from sub-tropical low-pressure
belt to sub-polar high-pressure belt.
Which of the above pairs are correctly matched? 2. They have considerable influence over
(a) 1 and 3 only British type of climate.

(b) 1 and 4 only 3. Westerlies in the southern hemisphere are


stronger and more constant in direction.
(c) 2 and 3 only
Which of the above statements are correct?
(d) 2 and 4 only
(a) 1 and 2 only
(b) 2 and 3 only
Q7. Consider the following statements about the
(c) 1 and 3 only
tropical and temperate cyclones:
(d) 1, 2 and 3
1. Tropical cyclones are elliptical in shape.
The shape of temperate cyclones is Q10. Which of the following is/are the reasons of a
'inverted V'. greater number of cyclones on the eastern
coast of India than on western coast?
2. Both tropical and temperate cyclones can be
1. Relative temperature difference between Bay
so wide that they can cover an area of Bengal and Arabian Sea.
equivalent to Europe.
2. Typhoons of Pacific Ocean
3. Tropical cyclones can originate over land 3. Drainage of rivers
and sea, but temperate cyclones can Select the correct answer using the code given
originate only over land. below:
Which of the above statement is/are correct? (a) 1 only

(a) 1 only (b) 1 and 2 only


(c) 2 and 3 only
(b) 1 and 3 only
(d) 1, 2 and 3
(c) 2 and 3 only

(d) 3 only Q11. Which of the following statements about


Southwest Monsoon in India is correct?

63
1. Both Indian Ocean Dipole and El-Nino Q14. Consider the following statements about
have adverse effects on Southwest Western Disturbances:
Monsoon. 1. Western Disturbances move towards
2. 40% of the country’s annual rainfall is northwest India under the influence of
received from Southwest Monsoon. Tropical Easterly Jet Stream.
3. In Maharashtra, Telangana and Andhra 2. They usually get moisture from
Pradesh, the date of onset of Southwest Mediterranean and Atlantic Ocean.
Monsoon is the same. 3. High pressure over Ukraine helps in
Which of the above statement is/are correct? formation of Western Disturbances.
(a) 1 only Which of the above statements is/are correct?
(b) 1 and 2 only (a) 1 only
(c) 3 only (b) 1 and 2 only
(d) None of the above (c) 2 and 3 only
(d) 3 only
Q12. Consider the following statements about
Hadley Cell: Q15. Which of the following statements with
1. Solar insolation is the reason for initiation of reference to Omega Block is correct?
Hadley cell.
(a) It is a low-pressure pattern that blocks and
2. Because of its location near the Intertropical
diverts jet streams.
Convergence Zone, its vertical extent is up
to the stratosphere. (b) Because of the Omega block, cold air from
3. The Hadley cell is one of the reasons for the southern Europe and Africa can be pulled
existence of the Namib desert. north.
Which of the above statements is/are correct? (c) Formation of Omega block leads to
(a) 1 only formation of Spanish plume.
(b) 1 and 2 only (d) None of the above
(c) 1 and 3 only
(d) 2 and 3 only Q16. Consider the following statements about
effects of El-Nino:
Q13. Consider the following statements about 1. It results in dry conditions and forest fires
Tropical cyclones: in Australia and Indonesia.
1. The eye of a tropical cyclone is a region of 2. Fewer hurricanes over the Atlantic Ocean,
calm and subsiding warm air.
the Caribbean Sea and the Gulf of Mexico.
2. Tropical cyclones generally do not have
lightning. 3. Bleaching of corals in the Lakshadweep
islands.
Which of the above statements is/are correct?
Which of the above statements are correct?
(a) 1 only
(b) 2 only (a) 1 and 2 only

(c) Both 1 and 2 (b) 2 and 3 only


(d) Neither 1 nor 2 (c) 1 and 3 only
(d) 1, 2 and 3

64
Q20. The pattern of planetary winds depends upon
Q17. Consider the following statements about which of the following factors?
walker circulation: 1. Emergence of Pressure belts
1. Walker circulation is formed due to 2. Rotation of earth
development of pressure gradient from west 3. Coriolis force
to east in the equatorial Pacific Ocean. Select the correct answer using the code given
2. Years of walker circulation are beneficial for below:
the people of Peru. (a) 1 and 2 only
3. La-Nina is a strengthened phase of walker (b) 2 only
circulation. (c) 2 and 3 only
(d) 1, 2 and 3
Which of the above statements are correct?
(a) 1 and 2 only
Q21. Consider the following statements about Dew
(b) 2 and 3 only formation:
(c) 1 and 3 only 1. In the process of Dew formation, it is
(d) 1, 2 and 3 necessary that the dew point is above the
freezing point.
2. The ideal conditions for the formation of
Q18. Which of the following is responsible for white frost are the same as those for dew
bringing abrupt changes in temperature and formation.
pressure and causing the air to rise to form
Which of the statements given above is/are correct?
clouds and cause precipitation?
(a) 1 only
(a) Coriolis Force (b) 2 only
(b) Geostrophic winds (c) Both 1 and 2
(c) Occluded Fronts (d) None of the above
(d) Pressure Gradient Force

Q22. Consider the following statements regarding


Q19. Under which of the following Conditions EL-NINO:
Tropical cyclones are formed and intensified? 1. The warm water of the central Pacific
1. Presence of Coriolis force replaces the cool Peruvian current.
2. Larger Variations in the vertical wind speed 2. Heavy rainfall is seen along the arid west
3. Large supply of moist air coast of South America.
4. Local disturbances 3. During El nino, the usual upwelling of
cold, nutrient-rich deep ocean water is
Select the correct answer using the code given
significantly reduced.
below:
Which of the statements given above is/ are
(a) 1 and 4 only
correct?
(b) 1, 3 and 4 only
(a) 1 and 2 only
(c) 1 and 2 only
(b) 1 only
(d) 1, 2, 3 and 4
(c) 1 and 3 only
(d) 1, 2 and 3

65
Q23. With reference to the Convectional rainfall, 3. Western coasts in the cool temperate zone.
Consider the following statements: Which of the statements given above is/are correct?
1. Such rains are common in the hotter part of (a) 1 only
the day. (b) 1 and 3 only
2. Such rains occur when a saturated air mass (c) 3 only
comes across a mountain, it is forced to (d) 1, 2 and 3
ascend and as it rises and expands,
temperature falls.
Q27. Under which of the following Conditions, the
3. Such rains occur in the interior of continents process of Condensation takes place:
and in the equatorial regions.
1. When the temperature of the air reduces to a
Which of the statements given above is/are correct? dew point with constant volume.
(a) 1 only 2. By adding moisture to air through
(b) 1 and 2 only evaporation.
3. When both volume and temperature are
(c) 1 and 3 only
reduced.
(d) 1, 2 and 3
Which of the statements given above is/are correct?
(a) 1 only
Q24. Consider the following statements regarding
local winds: (b) 1 and 2 only
(c) 2 and 3 only
1. Sea breeze flows over the land during the (d) 1, 2 and 3
night, Whereas the land breeze flows during
the day time.
2. Differences in the heating and cooling of Q28. Consider the following Pairs:
earth surfaces is one of the factors
responsible for formation of local or Oceans/Areas Tropical Storms
regional winds. 1. Western Australia Willy - willies
Which of the statements given above is/are correct? 2. Western Pacific Hurricanes
(a) 1 only 3. Atlantic Typhoons
(b) 2 only Which of the pairs given above is/are correctly
(c) Both 1 and 2 matched?
(d) None of the above
(a) 1 and 2 only
(b) 1 only
Q25. Which of the following statements is incorrect (c) 2 and 3 only
about Extra Tropical Cyclones?
(d) 1, 2 and 3
(a) Extra tropical cyclones are formed along the
polar front.
(b) Extra tropical cyclone affects a smaller area Q29. Consider the following statements regarding
i.e., its impact is less. Thunderstorms:
(c) Their movement is from West to east which
is opposite to tropical cyclones. 1. They occur over a small area but are
(d) All the above statements are correct. violent in nature
2. A thunderstorm is a well-grown
cirrocumulus cloud producing thunder and
Q26. Which of the following areas receive heavy
lightning.
rainfall of over 200 cm per annum?
1. The equatorial belts.
2. The coastal areas of the continents.

66
3. When the clouds extend to heights here
sub-zero temperature prevails, thails are
formed and they come down as hailstorms.
Which of the statements given above is/ are
correct?
(a) 1 and 3 only
(b) 1 and 2 only
(c) 1 only
(d) 1, 2 and 3

Q30. Consider the following statements regarding


Coriolis force:
1. Coriolis force is directly proportional to the
angle of latitude.
2. Coriolis force affects wind velocity and
wind direction.
3. Coriolis force is absent at the equator and
maximum at the poles.
Which of the statements given above is/are correct?
(a) 1 and 2 only
(b) 3 only
(c) 1 and 3 only
(d) 1, 2 and 3

Answer key
1. Ans: D 16. Ans: D
2. Ans: C 17. Ans: B
3. Ans: A 18. Ans: C
4. Ans: B 19. Ans: B
5. Ans: C 20. Ans: D
6. Ans: C 21. Ans: C
7. Ans: A 22. Ans: D
8. Ans: C 23. Ans: C
9. Ans: B 24. Ans: B
10. Ans: D 25. Ans: B
11. Ans: C 26. Ans: B
12. Ans: C 27. Ans: D
13. Ans: C 28. Ans: B
14. Ans: C 29. Ans: A
15. Ans: C 30. Ans: C

67
World climate
Q1. Consider the following statements regarding (b) North-Eastern Monsoon brings rainfall to
the Equatorial climate: Northern India during the winter season.
1. They exhibit a layered arrangement of (c) There is decline in the monsoonal rains in
vegetation with dense undergrowth. the west to east direction.
2. The winter season in the equatorial climates (d) Humidity is more pronounced in the regions
is generally devoid of rains. of equatorial climate as against monsoonal
Which of the statements given above is/are climate.
incorrect?
(a) 1 only Q4. With respect ot polar climates, consider the
(b) 2 only following statements:

(c) Both 1 and 2 1. The continuous sunshine during the


summer greatly raises the air temperature of
(d) Neither 1 nor 2
the Polar regions.
2. Declining ice cover over the Arctic is
Q2. Consider the following statements regarding compounded by the positive ice albedo
Climate: cycle.
1. Land and sea breezes over a vast Which of the statements given above is/are correct?
geographical area characterize the
(a) 1 only
monsoonal climate.
(b) 2 only
2. Shifting cultivation and livestock farming
are actively practiced in the equatorial (c) Both 1 and 2
regions. (d) Neither 1 nor 2
3. Savanna is characterized by the presence
of the deciduous trees.
Q5. The climate is characterized by warm and dry
Which of the statements given above is/are correct? summer while most of the rainfall occurs in
(a) 1 and 3 only winter months. The area is flanked by the
presence of both hot and cold local winds
(b) 2 only
adorned by a variety of orchards.
(c) 2 and 3 only
Which of the following climatic regions is best
(d) 1, 2 and 3 described by the above description?
(a) Steppes
Q3. Which among the following statements is most (b) Sudan
appropriate regarding the Monsoon climate?
(c) Tropical Monsoon
(a) More than half of the rainfall occurs in the
(d) Mediterranean
Tamil Nadu during the month of August.

68
Q6. Consider the following statements regarding Q9. Consider the following statements regarding
Equatorial and Tropical climates: Deserts:
1. Equatorial forests lie below the equator in 1. Deserts in South America are formed only
South America. because of Continentality.
2. Tropical marine climates are similar to the 2. Western boundaries of most African deserts
tropical monsoon climates in terms of are flanked by cold ocean currents.
rainfall patterns. 3. Desert climates are favorable around the
Which of the statements given above is/are correct? subtropical high-pressure belts.

(a) 1 only Which of the statements given above is/are correct?


(b) 2 only (a) 1 and 2 only

(c) Both 1 and 2 (b) 2 and 3 only

(d) Neither 1 nor 2 (c) 3 only


(d) 1, 2, and 3
Q7. Consider the following statements about
grasslands: Q10. Consider the following statements regarding
1. Equator divides the Llanos and Campos grasslands:
grasslands of Savanna. 1. The temperate grasslands of South
America are devoid of marine influence.
2. Westerlies are dominant in shaping the
climate of Savanna regions. 2. Steppes in the Northern Hemisphere
experience mild winters due to
Which of the statements given above is/are correct?
continentality.
(a) 1 only Which of the statements given above is/are correct?
(b) 2 only (a) 1 only
(c) Both 1 and 2 (b) 2 only
(d) Neither 1 nor 2 (c) Both 1 and 2
(d) Neither 1 nor 2
Q8. Consider the following statements regarding
agricultural and allied sector: Q11. Consider the following statements regarding
1. Rice cultivation accounts for maximum Himalayan Ecosystem:
Greenhouse gases emissions from the 1. Valley of flowers national park forms the
agricultural and allied sector. core area of the Nanda Devi Biosphere
reserve.
2. Areas with monsoonal climate largely
depend upon agriculture for livelihood. 2. Glacial retreats are an important cause of
the Glacial Lake Outburst floods.
Which of the statements given above is/are correct?
3. The National Mission for sustaining
(a) 1 only Himalayan Ecosystem is under the
(b) 2 only Ministry of Environment, Forests, and
Climate Change.
(c) Both 1 and 2
Which of the statements given above are correct?
(d) Neither 1 nor 2

69
(a) 1 and 2 only (c) Both 1 and 2
(b) 2 and 3 only (d) Neither 1 nor 2
(c) 1 and 3 only
(d) All of the above Q15. Consider the following statements about
Tundra regions:

Q12. Consider the following pairs: 1. Coniferous trees are totally absent below
the Equator due to the narrowness of
Grasslands Country
continents.
1. Llanos Ecuador
2. Laurentian climate finds presence in the
2. Pampas Uruguay Northern Hemisphere only.
3. Pustaz Hungary Which of the statements given above is/are correct?
4. Canterbury Britain (a) 1 only
Which of the pairs given above are correctly (b) 2 only
matched?
(c) Both 1 and 2
(a) 1 and 2 only
(d) Neither 1 nor 2
(b) 2 and 3 only
(c) 1 and 4 only
Q16. Consider the following pairs of grasslands and
(d) 1 and 3 only
climate:
1. Land of big games Savanna
Q13. Consider the following statements regarding
2. Granaries of the world Prairies
different climates:
3. Cold pole of the Earth Siberia
1. Temperate Monsoon climates are mostly
found in China. Which of the above pairs are correctly matched?
2. Gulf type climate is most prominent along (a) 1 and 2 only
the coast of Yemen. (b) 2 and 3 only
Which of the statements given above is/are correct? (c) 1 and 3 only
(a) 1 only
(d) All of the above
(b) 2 only
(c) Both 1 and 2 Q17. Consider the following statements regarding
(d) Neither 1 nor 2 Arctic region:
1. Arctic Council is a non-governmental
organization aimed at environmental
Q14. Consider the following statements regarding
protection in the Arctic Region.
Tundra region and MOSAiC project:
1. The whole of Tundra region is 2. Himadri is India’s research base deployed
characterized by permanent snow cover. in Norway for the study of the region.

2. MOSAiC project is designed for the study Which of the statements given above is/are correct?
of the Antarctica climate. (a) 1 only
Which of the statements given above is/are correct? (b) 2 only
(a) 1 only (c) Both 1 and 2
(b) 2 only (d) Neither 1 nor 2

70
(b) 2 only
Q18. Consider the following statements regarding (c) Both 1 and 2
Kyoto Protocol and Greenhouse gases:
(d) Neither 1 nor 2
1. Greenhouse gases are capable of absorbing
short-wave radiations of the sun.
2. Both the Kyoto Protocol and Paris Q21. Consider the following statements:
Agreement impose legally binding 1. Lake Priyadarshini is built by India around
obligations. Maitri research base, Antarctica.
3. India aims to have 70% of non-fossil fuel- 2. Polar Vortex and Polar Stratospheric clouds
based electricity by 2030.
are directly responsible for Ozone depletion.
Which of the statements given above is/are correct?
3. Kigali Amendment to the Montreal Protocol
(a) 1 and 2 only is legally binding in nature.
(b) 2 only Select the correct answer using the codes given
(c) 1 and 3 only below:
(d) 3 only (a) 1 and 2 only
(b) 1 and 3 only
Q19. Consider the following about nitrous oxide (c) 2 only
(N2O) emission:
(d) 1,2 and 3
1. Neem coating of urea
2. Soil Health card
Answer key
3. Organic fertilizers
1. Ans: C 12. Ans: B
Which of the above is/are useful in reducing 2. Ans: A 13. Ans: A
the emission of nitrous oxide (N2O)? 3. Ans: D 14. Ans: D
(a) 1 and 2 only 4. Ans: B 15. Ans: B
5. Ans: D 16. Ans: D
(b) 1, 2, and 3 6. Ans: D 17. Ans: C
(c) 3 only 7. Ans: A 18. Ans: B
8. Ans: B 19. Ans: A
(d) 2 and 3 only 9. Ans: B 20. Ans: C
10. Ans: D 21. Ans: D
11. Ans: A
Q20. Consider the following statements regarding
Steppes and Thermal expansion:
1. Steppes are found in the areas around the
Black Sea and the Caspian Sea.
2. Thermal expansion of the ocean water is the
main contributing factor to the rise in sea
level.
Which of the statements given above is/are correct?
(a) 1 only

71
India and location
Q1. Consider the following statements regarding 3. Duar formations in the Sikkim region is an
International borders: extension of the Shivalik.
1. Mizoram and Nagaland share a border with Which of the statements given above are correct?
Myanmar only.
(a) 1 and 3 only
2. Sikkim is the only state forming an
(b) 2 and 3 only
international boundary on the three sides.
(c) 1, 2 and 3
3. Tripura forms the longest boundary with
Bangladesh. (d) None of the above

Which of the statements given above is/are


incorrect? Q4. Consider the following peaks:
(a) 3 only 1. Saramati
(b) 2 and 3 only 2. Nokrek
(c) 2 only 3. Makalu
(d) All of the above 4. Annapurna
5. Kamet
Q2. Consider the following pairs about points of 6. Nanda Devi
India and locations: Which of the following is the correct sequence of
1. Indira col Andaman and the above peaks in the east to west direction?
Nicobar Island (a) 2-1-4-3-5-6
2. Kibithu Nagaland
(b) 1-2-3-4-6-5
3. Guhar Moti Gujarat
(c) 2-1-4-3-6-5
Which of the above pairs is/are correctly matched?
(d) 1-2-4-3-6-5
(a) 1 only
(b) 1 and 2 only
Q5. Consider the following statements regarding
(c) 1 and 3 only mountain ranges in India:
(d) 3 only 1. Aravallis are spread across the four Indian
states and UTs.

Q3. Consider the following statements regarding 2. Himalayas do not extend to the state of
Tripura.
Hill Ranges:
3. The tree line of the western Himalayas is
1. The Himalayas in the North East India are
less than the Eastern.
continuously aligned in the North to South
direction. Which of the statements given above are correct?
(a) 1 and 3 only
2. Meghalaya and the Karbi Anglong hills are
the extension of Deccan Plateau. (b) 1 and 2 only

72
(c) 2 and 3 only Select the correct answer using the codes given
(d) All of the above below:

Q6. Consider the following statements regarding (a) 1 and 2 only


Karewas and Bugyals: (b) 2 and 4 only
1. Karewas are confined to Jammu and (c) 1 and 3 only
Kashmir while Duns are confined to (d) 1 and 4 only
Uttarakhand.
2. Bugyals are the grasslands located in the
valleys of Uttarakhand. Q9. Consider the following statements regarding
Select the correct answer using the codes given alluvial plains:
below: 1. Bhabar areas are more suitable for crop
(a) 1 only cultivation than Terai.
(b) 2 only 2. Khadar sediments are deposited near the
(c) Both 1 and 2 flood plains.
(d) Neither 1 nor 2 Which of the statements given above is/are correct?
(a) 1 only
Q7. Consider the following statements regarding (b) 2 only
Indian Standard Meridian:
(c) Both 1 and 2
1. The Indian Standard Meridian (ISD) cuts
across the centre of Andhra Pradesh. (d) Neither 1 nor 2

2. The smallest part of ISD falls in the state of


Madhya Pradesh. Q10. Consider the following statements regarding
alluvial plains:
Select the correct answer using the codes given
below: 1. Bhabar, Tarai, Khadar, and Bhangar
together constitute alluvial plains.
(a) 1 only
2. Most of the Brahmaputra plains lie in the
(b) 2 only state of Arunachal Pradesh.
(c) Both 1 and 2 Which of the statements given above is/are correct?
(d) Neither 1 nor 2 (a) 1 only
(b) 2 only
Q8. Consider the following statements regarding (c) Both 1 and 2
Molassis basin and Mountain ranges: (d) Neither 1 nor 2
1. Molassis basin is an important feature in the
state of Mizoram. Q11. Consider the following statements regarding
2. Mahabharat range is located between the Ghats and Plateaus:
Indo-Nepal border. 1. The elevation of western ghats increases
southwards.
3. Aksai Chin forms a part of the Karakoram
range. 2. Eastern ghats are continuous with lower
elevation.
4. Nubra valley is accessed through the
3. Bundelkhand forms the upper part of the
Khardung La pass.
Deccan Plateau.

73
Which of the statements given above is/are correct? 2. This area shows opposing slopes in the
(a) 1 and 2 only Northern and the Southern parts.

(b) 1 only Which of the statements given above is/are correct?

(c) 2 and 3 only (a) 1 only

(d) 1, 2 and 3 (b) 2 only


(c) Both 1 and 2

Q12. Consider the following statements regarding (d) Neither 1 nor 2


mountain ranges and faults:
1. All the rivers of the Central Highlands Q15. Consider the following statements regarding
originate in the Vindhya and Satpura the Indian coasts:
ranges. 1. Natural harbors and ports are a
2. Malda fault exists between the Malwa characteristic feature of the emergent
Plateau and the Shillong Plateau. coasts.
Which of the statements given above is/are 2. Gujarat has the maximum coastline
incorrect? amongst all the States/UTs of India.
(a) 1 only 3. Northern Circars are the region lying
(b) 2 only between Mahanadi and Krishna

(c) Both 1 and 2 Which of the statements given above is/are correct?

(d) Neither 1 nor 2 (a) 1 and 2 only


(b) 3 only

Q13. Consider the following ranges/mountains: (c) 2 and 3 only

1. Harishchandra (d) 1, 2 and 3

2. Nallamala
3. Dandakaranya Q16. Which of the following statements is most
4. Ramgarh appropriate regarding the Islands of India?

5. Malya giri (a) The entire Island of Lakshadweep lies


below the 12-degree channel.
6. Garhjat
Arrange the above ranges in the south-north (b) Duncan Passage is situated between
direction: Swaraj dweep and the Shaheed dweep.
(a) 2-3-1-6-5-4 (c) Agatti Island is situated to the north of the
(b) 2-1-3-4-6-5 capital Kavaratti.

(c) 2-1-3-5-6-4 (d) The highest peak of Andaman, Saddle


(d) 1-2-4-3-6-5 Peak is located in the South Andaman.

Q14. Consider the following statements regarding Q17. Consider the following statements regarding
the Indian desert: Andaman and Nicobar Islands:
1. Absence of marine deposits indicates that 1. These islands are an extension of the
the region was a part of the Peninsular Arakan Yoma range.
plateau.
2. Narcondam Island in the Andaman is a
dormant volcano.

74
3. Mt. Thullier is the highest peak of the
Andaman group of Islands.
Which of the statements given above are correct?
(a) 1 and 3 only
(b) 1 and 2 only
(c) 2 and 3 only
(d) 1, 2 and 3

Q18. Consider the following statements regarding


Lakshadweep:
1. Andrott is the largest group of Islands in
Lakshadweep.
2. Minicoy Island is separated from the rest of
the island through a Nine-degree channel.
Which of the statements given above is/are correct?
(a) 1 only
(b) 2 only
(c) Both 1 and 2
(d) Neither 1 nor 2

Answer Key
1. Ans: D 10. Ans: D
2. Ans: D 11. Ans: B
3. Ans: D 12. Ans: C
4. Ans: B 13. Ans: C
5. Ans: A 14. Ans: B
6. Ans: D 15. Ans: B
7. Ans: B 16. Ans: C
8. Ans: D 17. Ans: B
9. Ans: B 18. Ans: C

75
Drainage
Q1. Consider the following statements regarding Q4. Consider the following statements regarding
rivers: Ganga river:
1. Peninsular rivers have greater drainage 1. Joshimath is associated with the river
basins due to their fixed course. Bhagirathi.
2. Himalayan rivers exhibit a single drainage 2. The length of Ganga is least in the state of
pattern throughout their course. Uttarakhand.
Which of the statements given above is/are correct? 3. The Ganga River system is entirely
(a) 1 only composed of rivers of Himalayan origin.

(b) 2 only Which of the statements given above is/are correct?

(c) Both 1 and 2 (a) 1 and 3 only

(d) Neither 1 nor 2 (b) 2 only


(c) 3 only
Q2. Consider the following events: (d) 1 and 2 only
1. Development of Malda gap
2. Upliftment of Potwar Plateau Q5. Consider the following pairs of Rivers and
Source:
3. Upliftment of western Himalayas
Rivers Source
4. Emergent Western coasts
1. Alaknanda Satopanth Glacier
Which of the above events are responsible for the
evolution of present-day Himalayan rivers? 2. Yamuna Bandarpunch

(a) 1, 2 and 4 only 3. Son Rajmahal hills

(b) 1, 2 and 3 only 4. Kosi Dhaulagiri

(c) 2, 3, and 4 only Which of the above pairs are correctly matched?

(d) 1, 3, and 4 only (a) 1 and 4 only


(b) 2 and 3 only
Q3. Consider the following statements regarding (c) 1 and 2 only
Indus River system: (d) 2 and 4 only
1. Jhelum river finds its origin in India itself.
2. Ravi river finds its origin in Himachal Q6. Which of the following National Parks are in
Pradesh. the vicinity of river Brahmaputra?
3. Indus drains the UTs of Jammu Kashmir and 1. Kaziranga National Park
Ladakh. 2. Pobitora wildlife Sanctuary
Which of the statements given above are correct? 3. Nameri National Park
(a) 1 and 2 only Select the correct answer using the codes given
(b) 2 and 3 only below:
(c) 1 and 3 only (a) 1 and 2 only
(d) All of the above (b) 2 and 3 only
(c) 1 only

76
(d) 2 only
Q10. Consider the following pairs:
Q7. Consider the following statements regarding Rivers State of Origin
Catchment area and Drainage basin: 1. Mahanadi Odisha
1. Catchment area of a river is located near the 2. Kaveri Kerala
source of the river.
3. Tapi Gujarat
2. Drainage basin of a river is formed by the
4. Krishna Karnataka
mainstream excluding the tributaries.
Which of the above pairs are correctly matched?
Which of the statements given above is/are
incorrect? (a) 1, 2 and 4 only
(a) 1 only (b) 1, 3 and 4 only
(b) 2 only (c) 3 and 4 only
(c) Both 1 and 2 (d) None of the above
(d) Neither 1 nor 2
Q11. Consider the following statements:
Q8. Consider the following rivers: 1. Vamsadhara is a west flowing river
1. Sindh forming an estuary.

2. Son 2. Andhra Pradesh, Karnataka, Kerala forms


the drainage basin of Pennar river.
3. Mayurakshi
Which of the statements given above is/are
4. Ken
incorrect?
Arrange the rivers above in the west to east
(a) 1 only
direction:
(b) 2 only
(a) 3-1-4-2
(c) Both 1 and 2
(b) 1-4-2-3
(d) Neither 1 nor 2
(c) 3-4-1-2
(d) 1-2-4-3
Q12. Consider the following statements regarding
Himalayan rivers and Harike wetland:
Q9. Consider the following statements regarding 1. The discharge of the Himalayan rivers is
drainage systems:
constant throughout the year.
1. Antecedent streams maintain their course
2. Harike wetland is situated at the
despite the upliftment of the surrounding
confluence of Satluj and Beas.
surface.
Which of the statements given above is/are correct?
2. Consequent streams flow according to the
slope of the adjoining surface. (a) 1 only

Which of the statements given above is/are correct? (b) 2 only

(a) 1 only (c) Both 1 and 2

(b) 2 only (d) Neither 1 nor 2

(c) Both 1 and 2


(d) Neither 1 nor 2

77
Q13. Consider the following rivers: Q16. Which of the following statement is correct?
1. Raidak (a) Majority of Indian rivers flow through a rift
2. Rind valley.

3. Sengar (b) Lacustrine deposits have been found from


the Shivalik.
4. Sindh
(c) There is a sharp tilt of the Indian peninsula
5. Son
from west to east.
Which of the above are the tributaries of Yamuna?
(d) Most of Tapi’s drainage basin lies in the state
(a) 1,2, and 4 only of Gujarat.
(b) 2, 3 and 4 only
(c) 3, 4, and 5 only Q17. With reference to river Tapi, consider the
following statements:
(d) 1, 4, and 5 only
1. It originates from the Amarkantak plateau.
2. Purna and Panjhra are important tributaries
Q14. Consider the following statements regarding of river Tapi.
drainage pattern: 3. It flows through a rift valley.
1. Rivers originating from the Amarkantak 4. It flows exclusively in Madhya Pradesh and
range are an example of centripetal drainage. Gujarat.
2. Rivers Narmada and Son flow in different Which of the statements given above are correct?
directions despite the same source of origin.
(a) 1 and 2 only
Which of the statements given above is/are correct? (b) 1, 2 and 4 only
(a) 1 only (c) 2 and 3 only
(d) 2, 3 and 4 only
(b) 2 only
(c) Both 1 and 2
Q18. Which of the following is/are the left
(d) Neither 1 nor 2 tributaries of Indus?
1. Shyok
Q15. Consider the following south Indian rivers: 2. Zaskar
1. Penganga
3. Chenab
2. Penneru
Select the correct answer using the code given
3. Tungabhadra below:
4. Manjira (a) 1 and 2 only
The correct arrangement of these rivers in the (b) 2 and 3 only
North-South direction is: (c) 1 and 3 only
(d) All the above
(a) 1-3-4-2
(b) 1-4-3-2
Q19. Arrange the following left bank tributaries of
(c) 4-1-3-2 Ganga from west to east:
(d) 4-1-2-3 1. Gandak
2. Gomti

78
3. Ramganga (b) 1 and 4 only
4. Ghaghara (c) 2 and 3 only
Select the correct answer using the code given (d) 3 and 4 only
below:
(a) 2-3-1-4
Q23. With reference to the geography of India,
(b) 2-4-1-3
which of the following are the west flowing
(c) 3-2-4-1
rivers?
(d) 3-4-2-1
1. Kalinadi
2. Periyar
Q20. Which of the following are the characteristics 3. Mahanadi
of the Peninsular Drainage System? 4. Palar
1. Senile topography Select the correct answer using the codes given
2. Antecedent drainage below:
3. Flow through shallow valleys
(a) 1 and 2 only
Select the correct answer using the code given (b) 1 and 3only
below: (c) 2 and 4 only
(a) 1 and 3 only (d) 3 and 4 only
(b) 1 and 2 only
(c) 2 and 3 only
Q24. Which of the following is/are the reasons for
(d) 1, 2 and 3
lack of formation of delta by peninsular rivers
falling into the Arabian sea?
Q21. With reference to the peninsular drainage 1. Presence of Hard rocks.
system, which of the statements are correct? 2. Presence of Steep slopes.
1. The Godavari is one of the largest 3. Flowing through faults.
peninsular river systems. Select the correct answer from the given codes
2. Its tributaries run through Maharashtra, below:
Madhya Pradesh, Orissa, etc.
3. Its important tributaries are penganga, (a) 1 only
Indravati and koyna. (b) 1, 2 and 3
(c) 3 only
Select the correct statement from the given code:
(d) 1 and 3 only
(a) 1 and 2 only
(b) 1 and 3 only
(c) 2 and 3 only Q25. Which of the following are projects on river
Chenab?
(d) All the above
1. Baglihar
Q22. Consider the following pairs: 2. Dulhasti
3. Salar
Indian Rivers Origin
4. Tulbul
1. Narmada -Amarkantak Plateau
Select the correct answer from the given codes
2. Godavari -Sahyadri Hills
below:
3. Krishna -Trimbakeshwar Plateau
(a) 1 and 2 only
4. Kaveri - Brahmagiri Hills
(b) 2 and 3 only
Which of the pairs given above are incorrectly (c) 1, 2 and 3 only
matched?
(d) 1, 2 and 4 only
(a) 1 and 2 only

79
Q26. Which of the following statements is /are Q29. Which of the following are an influence
correct about Luni? formation of delta?
1. It is the largest river system of Rajasthan. 1. Climate conditions.
2. It originates near Pushkar and the entire river 2. Sediment sources in the drainage basin.
system is ephemeral. 3. Tectonic stability.
Select the correct answer from the given codes 4. River slope and flooding characteristics.
below: Select the correct answer from the given codes
(a) 1 only below:
(b) 2 only (a) 1, 2 and 3 only
(c) Both 1 and 2 (b) 2 and 3 only
(d) None of the above (c) 1, 3 and 4 only
(d) All the above.
Q27. Which of the statements is/are incorrect
regarding drainage pattern?
Q30. With reference to the places and the
1. The drainage pattern resembling the confluence of rivers, which of the following
branches of a tree is known as trellis. pairs is incorrect?
2. When the primary tributaries of rivers flow a) Devprayag Alaknanda Bhagirathi
parallel to each other and secondary b) Rudraprayag Alaknanda Mandakini
tributaries join them at right angles, the
c) Vishnuprayag Alakhanada Dauliganga
pattern is known as dendritic.
3. When the rivers originate from a hill and d) Karnaprayag Alakhananda Nandan
flow in all directions, the drainage pattern is
known as radial.
Q31. Which of the following rivers is /are examples
Select the correct answer from the given codes of arcuate delta?
below: 1. The Nile
(a) 1 only 2. Ganges
(b) 1, 2 and 3 3. Mississippi
(c) 3 only Select the correct answer from the given codes
(d) 1 and 2 only below:
(a) 1 and 3 only
Q28. Which of the following is /are the important (b) 1, 2 and 3
pillars of the Namami gange programme? (c) 3 only
1. Sewerage Treatment Infrastructure. (d) 1 and 2 only
2. River-Front Development.
3. Biodiversity
Q32. Which of the following rivers forms the
Select the correct answer from the given codes Estuarine delta?
below. 1. Amazon
(a) 1 and 2 only 2. Ob
(b) 2 and 3 only
3. Ganga
(c) 1 and 3 only
(d) 1, 2 and 3 Select the correct answer from the given codes
below:

80
(a) 1 and 2 only
(b) 1, 2 and 3
(c) 2 only
(d) 1 and 3 only

Q33. Recently the Cabinet has given nod to the first


project under the National Perspective Plan
for interlinking of rivers. It envisages
transferring water from -
(a) Cauvery to Tungabhadra
(b) Godavari to krishna
(c) Ken to betwa
(d) Narmada to tapi

Answer Key
1. Ans: D 12. Ans: B 23. Ans: A
2. Ans: B 13. Ans: B 24. Ans: B
3. Ans: D 14. Ans: B 25. Ans: C
4. Ans: B 15. Ans: B 26. Ans: D
5. Ans: C 16. Ans: B 27. Ans: D
6. Ans: A 17. Ans: C 28. Ans: D
7. Ans: B 18. Ans: B 29. Ans: D
8. Ans: B 19. Ans: C 30. Ans: D
9. Ans: C 20. Ans: A 31. Ans: D
10. Ans: D 21. Ans: A 32. Ans: A
11. Ans: C 22. Ans: C 33. Ans: C

81
Indian climate
Q1. Which of the following parameters are 3. Sea Surface Temperature (SST of the
used by the Indian Meteorological equatorial South Indian Ocean.
Department (IMD) for the official
4. SST gradient between the North Atlantic and
declaration of the onset of Monsoon in
the North Pacific Oceans.
India?
Choose the Correct answer using the code given
1. Rainfall reported over various weather
below:
stations located in Kerala only
(a) 1,2 and 4 only
2. Atmospheric Pressure exerted by the
Westerlies (b) 2,3 and 4 only
3. Outwave/ Outgoing Longwave (c) 1 and 3 only
Radiation (d) All of the above
Choose the correct answer using the code
given below:
Q4. With reference to the Indian Ocean
(a) 1 and 2 only Dipole (IOD), consider the following
(b) 2 and 3 only statements:
(c) 1 and 3 only 1. It is characterized by an oscillation of
sea surface temperatures in which the
(d) 1,2 and 3
Western Indian Ocean becomes
alternately warmer and colder than the
Q2. Which of the following would have happened Eastern part of the Pacific Ocean.
if the Himalayas did not exist? 2. Unlike an El-Nino, the IOD is a
(a) There would be no North Eastern periodically recurring event and can be
Monsoon season experienced in India. predicted.

(b) The onset of monsoon would have taken 3. It is possible for IOD to cause floods in
place in the winter season. India and bushfires in Australia in the
same year.
(c) The Indian subcontinent would have
recorded a lower Long Period Average of Which of the above statements are correct?
rainfall received. (a) 1 and 2 only
(d) The Thar desert would have received very (b) 2 and 3 only
high rainfall every year.
(c) 3 only
(d) 1,2 and 3
Q3. Which of the following conditions are taken
into account by the Earth System Science
Organisation (ESSO and the India Q5. Which of the following are characteristic
Meteorological Department (IMD while features of the Indian Monsoons?
making forecasts about the monsoon in 1. Sudden Burst
India?
2. Inter season variability
1. Mean Sea Level pressure in east Asia.
3. Western Disturbances
2. The surface air temperature over northwest
4. Intra season variability
Europe.
5. Sudden withdrawal

82
Choose the correct answer using the code given (d) 1,2 and 3
below:
(a) 1,2 and 4 only
Q8. Which of the following statements is
(b) 3,4 and 5 only incorrect with reference to Cloudbursts?
(c) 1,2,3 and 4 only (a) The Specific location and time of a
(d) 1,2,3,4 and 5 cloudburst can be predicted only after the
genesis of a thunderstorm has already
begun.
Q6. Consider the following statements
regarding Indian monsoon: (b) These systems have a shorter life span
whereas large scale systems like tropical
1. The El-Nino is generally known to
cyclones have a longer life span.
suppress monsoon rains in India.
(c) A cloudburst can occur anytime and at any
2. The La-Nina has a negative impact on
place which is affected by convective
rainfall associated with the Northeast
weather systems.
monsoon.
(d) The India Meteorological Department
3. Heavy snowfall in Tibetan Plateau in
labels rainfall over 200 mm per hour as
the winter season can immensely
cloudburst.
increase the rainfall received in India
during the monsoon season.
Which of the above statements is/are correct? Q9. With reference to cyclones in India,
consider the following statements:
(a) 1 only
1. Historically in India, more cyclones
(b) 1 and 2 only
have impacted the Eastern coast in
(c) 1 and 3 only comparison to the Western Coast.
(d) 2 and 3 only 2. Historically in India, more cyclones
have occurred in the post monsoon
season in comparison to the Pre-
Q7. With reference to Western disturbances,
monsoon Season.
consider the following statements:
3. All cyclones that hit the Indian coast
1. These are weak tropical cyclones that
originate in the Indian Ocean.
originate over the Mediterranean Sea.
Which of the above statements are correct?
2. Jet streams play an important role in
driving these high-pressure systems (a) 1 and 2 only
into India. (b) 2 and 3 only
3. The rainfall that occurs due to these (c) 1 and 3 only
disturbances is known to be beneficial
(d) 1,2 and 3
for the Rabi crops.
Which of the above statements is/are
incorrect? Q10. With reference to Madden Julian
oscillations (MJO), consider the following
(a) 1 only
statements:
(b) 2 only
1. Unlike a standing pattern like the El
(c) 1 and 2 only Niño, the Madden–Julian oscillation is a

83
traveling pattern that propagates tea cultivated in Assam and the jute
eastward above the warm parts of the cultivated in West Bengal.
Indian and Pacific oceans.
Which of the above statements is/are
2. A MJO cycle can last anywhere between correct?
120 to 150 days.
(a) 1 only
3. A MJO can completely nullify the
impact of El-Nino on monsoon rains in (b) 1 and 2 only
India. (c) 2 and 3 only
Which of the above statements is/are (d) 1 and 3 only
correct?
(a) 1 only
Q13. According to the Indian Meteorological
(b) 1 and 2 only
Department, in which of the following
(c) 1 and 3 only states/union territories does the South
(d) 2 and 3 only West Monsoon arrive first?
(a) Kerala
Q11. With reference to heatwaves in India, (b) Lakshadweep
which of the following statements is
(c) Andaman and Nicobar Islands
incorrect?
(d) Tripura
(a) In General, heatwaves are defined with
reference to the normal maximum
temperature of a particular region. Q14. In India, The Western Ghats receive rain
(b) Most heat waves in India are formed over from which of the following sources on an
the Peninsular region especially on the annual basis?
leeward side of the Western ghats. 1. Bay of Bengal Branch of South West
(c) Inflow of Western disturbances in Monsoon
Northern India can influence the 2. Cyclones and depressions that originate
occurrence of heat waves in Central India. in the Bay of Bengal
(d) A cyclone hitting the cost of Odisha can 3. Cyclones and depressions that originate
trigger a heatwave in Punjab. in the Arabian Sea
4. Western Disturbances
Q12. Consider the following statements 5. North East Monsoons
regarding Norwesters: Select the correct answer using the code
1. Norwesters are violent storms that given below:
generally occur just before sunset or (a) 1, 3 and 5 only
just a few hours after it.
(b) 2, 3 and 5 only
2. In India, these storms originate over
(c) 2, 3 and 4 only
Chotanagpur Plateau only.
(d) 1, 2, 4 and 5 only
3. In India, the rainfall caused by
Norwesters is deemed harmful for the

84
Q15. With reference to the Inter Tropical Which of the statements given above is/are correct?
Convergence Zone (ITCZ), Which of the (a) 1 only
following statements is/are correct? (b) 2 only
1. ITCZ is a low pressure located at (c) Both 1 and 2
equator where trade winds are diverse. (d) Neither 1 nor 2
2. Coriolis force plays an important role
in changing the direction of wind from
Q18. Which of the following pairs is /are
southwest to northeast.
correctly matched?
Select the correct answer using the code
Pre-monsoon showers State
given below:
1. Mango shower -Kerala
(a) 1 only
2. Nor wester -Bengal and Bihar
(b) 2 only
3. Loo -Punjab
(c) Both 1 and 2
(d) None of the above Select the correct answer using the code given
below:
(a) 1 only
Q16. Which of the following are the
(b) 1, 2 and 3
characteristics of monsoon?
(c) 3 only
1. Rainfall received from the southwest
(d) 1 and 3 only
monsoons is permanent in character,
which occurs between June and
September. Q19. Which of the following statements is/are
2. The monsoon rainfall has a declining correct?
trend with decreasing distance from the 1. Climate changes quickly, may be within a
sea. day or week.
3. Topography plays an important role in 2. Weather changes imperceptibly and may
influencing monsoon rainfall. be noted after 50 years or even more.
Select the correct action from given codes Select the correct answer using the code
below: given below:
(a) 1 only (a) 1 only
(b) 1, 2 and 3 (b) 2 only
(c) 3 only (c) Both 1 and 2
(d) and 3 only (d) None of the above

Q17. With reference to monsoon in India, Q20. Which of the following statements is/are
consider the following statements: incorrect?
1. Entire southern India receives its major 1. The windward sides of Western Ghats and
portion of rainfall from retreating Assam receive high rainfall during June-
monsoon. September.
2. As we move from east to west the intensity 2. In winter months, the weather conditions
of rainfall decreases in northern plains. over India are generally influenced by the

85
distribution of pressure in Central and 1. Most parts of India receive rainfall in
Western Asia. the winter season.
2. Central parts of India and northern
Select the correct answer using the code
parts of the southern Peninsula also
given below. get winter rainfall occasionally.
(a) 1 only 3. Arunachal Pradesh and Assam in the
(b) 2 only northeastern parts of India have rains
during winter months.
(c) Both 1 and 2
(d) Neither 1 nor 2 Select the correct action from given codes
below:
(a) 1 only
Q21. Which of the following statements is/are (b) 1, 2 and 3
correct regarding western cyclonic (c) 1 and 3 only
disturbances? (d) 2 and 3 only
1. The western cyclonic disturbances are
brought into India by the easterly jet
Q24. Which of the following pairs is/are correct
stream.
vis a vis Koeppen scheme for classification
2. An increase in the prevailing night
temperature generally indicates an of climate?
advance in the arrival of these cyclone 1. Tropical climates—Where the mean
disturbances. monthly temperature throughout the year
is over 18°C.
Select the correct answer using the code
2. Warm temperate climates—Where mean
given below.
temperature of the coldest month is
(a) 1 only between 15°C and minus 4°C
(b) 2 only 3. Ice climates—Where mean temperature
(c) Both 1 and 2 of the warmest month is under 0°C to 10
(d) Neither 1 nor 2 °C
Select the correct action from given codes
Q22. Which of the following statements is/are below:
correct regarding the causes of the monsoon (a) 1 only
in the south asian region? (b) 1, 2 and 3
1. Formation of an intense low pressure in (c) 3 only
the northwestern part of the subcontinent. (d) 1 and 3 only
2. Formation of a low pressure in the Indian
Ocean in the south of the landmass.
Q25. Which of the following statements is/are
Select the correct action from given codes correct regarding the monsoon?
below:
1. Regional variations in monsoon climate
(a) 1 only help in growing various types of crops.
(b) 2 only 2. Sudden monsoon bursts create a problem
(c) Both 1 and 2 of soil erosion over large areas in India.
(d) Neither 1 nor 2 3. Agricultural prosperity of India depends
very much on timely and adequately
distributed rainfall.
Q23. Which of the following statements is/are
correct? Select the correct action from given codes
below:

86
(a) 1 only
(b) 1, 2 and 3
(c) 3 only
(d) 1 and 3 only

Answer Key
1. Ans: B 14. Ans: B
2. Ans: C 15. Ans: B
3. Ans: D 16. Ans: C
4. Ans: C 17. Ans: B
5. Ans: A 18. Ans: D
6. Ans: B 19. Ans: D
7. Ans: C 20. Ans: D
8. Ans: D 21. Ans: B
9. Ans: A 22. Ans: A
10. Ans: A 23. Ans: D
11. Ans: B 24. Ans: A
12. Ans: A 25. Ans: B
13. Ans: C

87
Natural Vegetation
Q1. Which among the following statements is Q4. Which of the following is/are reasons for the
correct about Black soils in India? soil degradation?
(a) Black soil only found in Madhya Pradesh 1. Wind
and Gujarat.
2. Water
(b) The black soils are generally clayey, deep
and impermeable. 3. High population
(c) The black soils are more suitable for tree 4. Deforestation
crops cashew nuts. Select the correct answer using the code given
(d) black soils are rich in nitrogen, potash, below.
magnesia and alumina.
(a) 1 and 3 only
(b) 2, 3 and 4 only
Q2. Consider the following statements regarding
(c) 1, 2 and 3 only
Laterite Soil in India:
(d) All of the above
1. Laterite Soils are more suitable for cashew
cuts.
2. Laterite Soils are widely cut as bricks for Q5. Consider the following statements with
house construction. reference to different types of soils:
3. They are poor in organic matter, Nitrogen, 1. Soils are usually thin on steep slopes and
phosphorus and calcium. thick over flat upland areas.
Which of the statements given above are 2. Tropical soils show shallow profiles than
correct? soils in frozen tundra regions.
(a) 1 and 2 only Which of the statements given above is/are
correct?
(b) 2 and 3 only
(a) 1 only
(c) 1 and 3 only
(b) 2 only
(d) All of the above
(c) Both 1 and 2
(d) Neither 1 nor 2
Q3. Arrange the following soil in the ascending
order of their availability in India:
1. Alluvial soil Q6. With reference to the red and yellow soils,
consider the following statements:
2. Red and yellow soil
1. They develop on crystalline igneous rocks in
3. Black soil
areas of high rainfall.
4. Laterite
2. The reddish color of the soil is due to the
Select the correct answer using the code given diffusion of Iron and when it occurs in a
below. hydrated form, it looks yellow.
(a) 4-3-2-1 Which of the statements given above is/are
(b) 2-1-4-3 incorrect?
(c) 1-2-3-4 (a) 1 only
(d) 2-3-1-4 (b) 2 only
(c) Both 1 and 2

88
(d) Neither 1 nor 2 2. It has optimum amount of Nitrogen and
Humus but is inadequate in potash.
3. Alluvial soils are loamier and more clayey in
Q7. They are generally sandy in structure and the lower and middle Ganga plain and the
saline in nature. Due to the dry climate, high Brahmaputra valley.
temperature and accelerated evaporation, Which of the statements given above is/are
they lack moisture and humus. It is correct?
characteristically developed in western
(a) 1 and 3 only
Rajasthan. These soils are poor and contain
little humus and organic matter. (b) 2 and 3 only

This is best description of: (c) 1 and 2 only

(a) Saline soil (d) All of the above

(b) Arid soil


(c) Laterite Soil Q10. In the context of duars, consider the following
statements:
(d) Forest soil
1. They are the alluvial floodplains in the
outer foothills of the Brahmaputra River
basin.
Q8. With the reference to the soils in India,
consider the following statements: 2. The duars have thin soil cover with high
inorganic content suitable for walnut
1. The soils of Indo-Gangetic-Brahmaputra cultivation.
plains India are formed by the
Which of the statements given above is/are
deterioration of rocks in-situ. correct?
2. The soils of the Peninsular plains are (a) 1 only
formed due to the depositional work of (b) 2 only
rivers and wind.
(c) Both 1 and 2
Which of the statements given above is/are (d) Neither 1 nor 2
incorrect?
(a) 1 only Q11. These cool temperate western margins are
(b) 2 only under the permanent influence of the
Westerlies all round the year. Summers are
(c) Both 1 and 2 never very warm here. The natural vegetation
(d) Neither 1 nor 2 of this climatic type is a deciduous forest like
oak, elm, ash, birch, beech, poplar, and
hornbeam.
Q9. Consider the following statements regarding Which of the following climatic types is described
to Alluvial Soil: in the passage given above?

1. They are the biggest soil group covering (a) British Type Climate.
about 40% of the total area of India. (b) Mediterranean Climate
(c) Steppe Climate

89
(d) North American climate (c) Both 1 and 2
(d) Neither 1 nor 2

Q15. Consider the following statements regarding


Q12. Consider the following statements regarding Tropical and semi evergreen forest:
equatorial climate: 1. They are found in wet and humid areas with
1. The equatorial vegetation comprises annual precipitation of over 200 cm.
evergreen trees that yield tropical hardwood. 2. There is a specific time for trees to shed their
2. The equatorial regions are generally densely leaves and flowering.
populated. 3. Semi evergreen forests have a mixture of
Which of the statements given above is/are evergreen and moist deciduous trees.
correct? Which of the statements given above is/are
(a) 1 only correct?

(b) 2 only (a) 1 and 2 only

(c) Both 1 and 2 (b) 2 only

(d) Neither 1 nor 2 (c) 3 only


(d) 1 and 3 only

Q13. With reference to Semi Evergreen Forests,


consider the following statements: Q16. Plants such as Tendu, palas, amaltas, bel,
1. The under growing climbers provide an khair, axlewood are the examples of which of
evergreen character to these forests. the following vegetation?

2. Semi Evergreen Forests are also called the (a) Tropical thorny forest
monsoon forests. (b) Tropical moist deciduous forest
Which of the statements given above is/are (c) Tropical Dry deciduous forest
correct?
(d) Semi Evergreen Forest
(a) 1 only
(b) 2 only
Q17. Which of the following conditions has led to
(c) Both 1 and 2
the establishment of plantations in the
(d) Neither 1 nor 2 equatorial regions?
1. Favorable hot and wet climate.
Q14. With reference to Himalayan vegetation, 2. Nutrient-rich tropical soils.
consider the following statements:
3. High temperature and rainfall
1. The Deciduous forests are found in the
4. Less problem of tree diseases.
foothills of the Himalayas.
2. Deodar is an endemic species of the western Select the correct answer using the code given
part of the Himalayan range. below.

Which of the statements given above is/are (a) 1, 2 and 4 only


correct? (b) 1 and 3 only
(a) 1 only (c) 1, 2 and 3 only
(b) 2 only (d) 2, 3 and 4 only

90
(c) 1 and 4 only
Q18. These thick forests occur in the regions near (d) All of the above
the equator and close to the tropics. The thick
canopies of the closely spaced trees do not
allow the sunlight to penetrate inside the forest Q21. Dolomite powder is sometimes applied in
even in the day time. agricultural lands. The purpose of applying it

This is best description of: is to:

(a) Temperate Evergreen Forest (a) Increase pH of the soil

(b) Tropical Deciduous Forest (b) Lower pH of the soil


(c) Tropical Evergreen Forest (c) Enrich biota in the soil
(d) Mediterranean Forest (d) Increase nitrogen content in the soil

Q19. Which of the following statements regarding


Q22. Consider the following National parks:
red soils of India are correct?
1. Campbell Bay National Park
1. They are generally poor in nitrogen and
humus. 2. Gugamal National Park
2. They are formed due to weathering of 3. Dandeli Anshi National Park
ancient crystalline granitic rocks.
4. Mukundara Hills National Park
3. Rice and pulses can be grown on these soils
and cotton cannot be grown on these soils. In which of the above National parks will you
find Ebony and Mahogany trees naturally?
4. They occupy roughly 40% of the total
geographical area of India. (a) 1 and 2 only

Select the correct answer using the code given (b) 1 and 3 only
below:
(c) 2 and 4 only
(a) 1 and 2 only
(d) 1,3 and 4 only
(b) 1,3 and 4 only
(c) 2 and 3 only
Q23. Which of the following methods is the most
(d) 1,2 and 4 only
appropriate for checking soil erosion on
Coastal and arid areas?
Q20. Consider the following states:
(a) Contour Bunding
1. Haryana
(b) Shelterbelts
2. Rajasthan
(c) Contour Ploughing
3. Gujrat
(d) Check Dams
4. Andhra Pradesh
Where will you naturally find “Thorny Scrub
Forests”? Q24. With reference to Tropical Moist Deciduous
(a) 1, 2 and 3 only Forests, which of the following statements
(b) 2 and 3 only is/are correct?

91
1. The Trees shed leaves their leaves annually (d) All of the above
during the autumn season.
2. In India, the total area covered by these Q27. Consider the following statements regarding
forests is much more than the area covered laterite soils of India:
by Littoral and Swamp Forests. 1. Laterite soils are generally red in colour.
3. In India, both South West monsoon and 2. Laterite soils are poor in nitrogen and
NorthEast Monsoon influence their growth. potash.
Choose the correct answer using the code given 3. Laterite soils are well developed in
below: Rajasthan and UP.

(a) 1 and 2 only 4. They are excellent for construction


purposes.
(b) 3 only
Which of the statements given above is/are correct?
(c) 2 and 3 only
(a) 1 only
(d) 1 and 3 only
(b) 2, 3 and 4
(c) 1 and 4 only
Q25. Which of the following forest types has the
(d) 1,2 and 4
highest carbon stock per hectare in India?
(a) Tropical Dry Deciduous Forests
(b) Himalayan Dry Temperate Forests
Answer Key
(c) Tropical Thorn Forests 1. Ans: B 15. Ans: C
(d) Tropical Wet Evergreen Forests 2. Ans: D 16. Ans: C
3. Ans: C 17. Ans: B
4. Ans: D 18. Ans: C
5. Ans: A 19. Ans: A
Q26. Consider the following statements:
6. Ans: A 20. Ans: D
1. Some of the states in India have both 7. Ans: B 21. Ans: A
Tropical Evergreen and Tropical Dry 8. Ans: C 22. Ans: B
Deciduous forests. 9. Ans: A 23. Ans: B
10. Ans: A 24. Ans: C
2. Some of the states in India have both 11. Ans: A 25. Ans: B
Tropical Evergreen and Mangrove Forests. 12. Ans: A 26. Ans: C
13. Ans: A 27. Ans: C
3. Some of the states in India have both 14. Ans: C
Tropical Evergreen forests and Alpine
Vegetation.
4. Every State in India has at least some
geographical area under Scrub Forests.
Which of the above statements are correct?
(a) 1 and 2 only
(b) 3 and 4 only
(c) 1,2 and 3 only

92
Agriculture
Q1. What are the advantages of Conservation Q3. Which of the following are used as Fertilizers
Agriculture? in the process of Fertigation?
1. Reduction of production and labor costs is 1. Ammonium Nitrate
possible.
2. Potassium Sulphate
2. It is possible to employ principles of
3. Phosphoric Acid
Conservation agriculture in a variety of
agricultural landscapes. 4. Urea

3. Increase in biodiversity is possible. 5. Monoammonium Phosphate

Select the correct answer using the code given Choose the correct answer using the code given
below:
below:
(a) 1, 2 and 5 only
(a) 1 and 3 only
(b) 2 and 3 only (b) 2, 3 and 4 only

(c) 1 only (c) 1, 2, 4 and 5 only

(d) 1, 2, 3, 4 and 5
(d) All of the above

Q4. With reference to the “Critical Period of Weed


Q2. What are the applications of allelopathy in
agriculture? Competition” in farming practices, consider
the following statements:
1. Cost of weedicides can be reduced as weed
1. It refers to the shortest time span during the
management is possible.
crop growth when weeding results in highest
2. Management and control of all insects and
economic returns.
pests is a possibility.
2. The crop yield level obtained by weeding
3. Reduction in development of yellow rust
during this period is almost similar to that
disease inWheat is possible.
obtained in a full season of weed free
4. Increased resistance towards abiotic stresses conditions.
is possible.
3. The critical period of weed competition is
Select the correct answer using the code given approximately half the duration of the crop.
below:
Which of the above statements is/are correct?
(a) 1, 2 and 3 only
(a) 1 and 2 only
(b) 1, 3 and 4 only
(b) 2 only
(c) 1, 2 and 4 only
(c) 2 and 3 only
(d) 2, 3 and 4 only
(d) All of the above

93
Q5. ‘Broadcasting’ refers to spreading fertilisers (a) 1, 2 and 3
uniformly all over the field. In this context, (b) 2 and 3 only
what could be the possible disadvantages of
(c) 1 and 2 only
the method of broadcasting?
(d) 2 only
1. Non-Mobile nutrients like Potassium cannot
be utilised at all by plant roots because they
move laterally over long distances. Q8. Which of the following most appropriately
2. Weed growth may be stimulated all over the describes the phenomenon of “Pesticide
field. Treadmill”?
3. Nitrogen losses due to denitrification may be (a) Biomagnification of pesticides in the tissues
enhanced. of tolerant organisms at successively higher
levels in a food chain.
Choose the correct answer using the code given
below: (b) Resistance in plant species due to erratic and
prolonged use of pesticide.
(a) 1 and 2 only
(c) Increase in doses of pesticides to prevent the
(b) 2 and 3 only
resurgence of pests that were being controlled
(c) 3 only with smaller doses earlier.
(d) All of the above (d) The sale or use of an adulterated, banned or
misbranded pesticide.
Q6. In India, ‘Metallic Lead’, ’Copper Sulphate’
and ‘Ethephon’ are viewed with Q9. Consider the following:
apprehension. These chemicals are used as:
1. Root & tuber crops
(a) Fruit ripening agents
2. Coconut
(b) Moisturising agents in cosmetics
3. Cocoa
(c) Preservatives in canned food
4. Coffee
(d) Weedicides in agriculture
5. Bamboo
6. Mushrooms
Q7. In the context of Indian agriculture, what are
Which of the above are included under the ‘Mission
the advantages of Foliar feeding of
for Integrated Development of Horticulture
Horticulture plants?
(MIDH)’?
1. It has proven to be an excellent method of
(a) 1,2 and 6 only
supplying secondary nutrients such as
calcium, magnesium, and sulfur to plants. (b) 1,3,4 and 5 only
2. It can avoid the problem of leaching of (c) 2,3,4 and 6 only
nutrients in soil. (d) 1,2,3,5 and 6 only
3. The nutrients can be mixed with pesticides,
thereby achieving a synergic effect and
additional saving on application costs.
Choose the correct answer using the code given
below:

94
Q10. “The crop is tropical in nature but can be (b) Genetic modification of crops to make them
grown in subtropical areas as well. It requires produce sterile seeds in the second generation.
well distributed rainfall of 50-100 centimeters (c) Genetic engineering of crops to produce an
for its growth. Curing is an important process enzyme that enables them to remain healthy
that makes the crop ready for the market. A even after being sprayed with glyphosate.
well-drained fertile sandy loam is suitable for
(d) None of the above
its growth.”
Which one of the following is the crop?
Q13. With reference to direct seeding of rice (DSR),
(a) Tobacco
consider the following statements:
(b) Tea
1. Direct-seeded rice matures 7 to 10 days
(c) Maize earlier than transplanted rice.
(d) Mustard 2. There is greater crop-weed competition in
DSR than that in transplanted rice.
Q11. A state in India has the following 3. The seed requirement for DSR is lower than
characteristics: that required in the transplantation method.
1. It is the largest producer of nutri cereals in Which of the above statements are correct?
India. (a) 1 and 2 only
2. The climate ranges from arid to semi-arid (b) 2 and 3 only
due to high variability in South Western
(c) 1 and 3 only
Monsoon rainfall.
(d) 1,2 and 3
3. The agrarian community relies on traditional
water harvesting structures such as Jhalras
and Khadeen. Q14. With reference to ‘Agri-business farming’,
Which of the following states has all the consider the following statements
features mentioned above? 1. It is a type of commercial farming managed
(a) Madhya Pradesh and financed by a group of small and
medium farmers who pool their lands and
(b) Rajasthan
resources.
(c) Haryana
2. It is highly mechanized and relies on
(d) Tamil Nadu chemicals and fertilizers.
Which of the statements given above is/are
Q12. Which of the following describes the correct?
phenomenon of ‘Terminator Technology’, (a) 1 only
sometimes seen in the context of Transgenic
(b) 2 only
crops?
(c) Both 1 and 2
(a) A gene editing technology that completely
eliminates mutation of disease-causing (d) None of the above
viruses in plants and helps overcome
challenges in breeding.

95
Q15. Consider the following statements regarding Select the correct answer using codes given
Forage crops: below:
1. Vegetative part of Forage crop is used as (a) 1 only
green fodder. (b) 2 and 3 only
2. Lemon grass, citronella grass, henna, and (c) 2 only
Japanese mint are some of the examples of
(d) 1 and 3 only
Forage crops.
Which of the statements given above is/are
correct? Q18. With reference to mixed farming, which of the
following statements is/are correct?
(a) 1 only
1. Mixed farming is characterized by low
(b) 2 only
capital expenditure on agricultural inputs.
(c) Both 1 and 2
2. Fodder crops are an important component of
(d) Neither 1 nor 2 mixed farming.

Select the correct answer using the code given


Q16. Which among the following are the uses of below.
seaweeds? (a) 1 only
1. Food for humans (b) 2 only
2. Potential indicators of pollution (c) Both 1 and 2
3. Used in treatment of Goitre (d) Neither 1 nor 2
Select the correct answer using codes given
below:
Q19. Consider the following statements:
(a) 1 only
1. The term 'Plankton' includes only the
(b) 1 and 2 only microscopic plants and not the microscopic
(c) 2 and 3 only animals.
(d) All of the above 2. Cyanobacteria and dinoflagellates are
examples of Phytoplankton.

Q17. With reference to Macronutrients Nitrogen, Which of the statements given above is/are
Phosphorus and potassium which of the incorrect?
following statements is/are correct? (a) 1 only
1. Ideal ratio considered for Nitrogen, (b) 2 only
Phosphorus and Potassium stands at 6:3:2. (c) Both 1 and 2
2. Phosphorus forms an integral part of nucleic (d) Neither 1 nor 2
acids that are carriers of genetic information.

3. Phosphorus ensures transportation of


Q20. With reference to organic fertilizers in India,
Photosynthesis throughout the plant.
Consider the following statements:

96
1. Fertilizer response ratio has been declining Which of the statements given above is/are
over time. correct?
2. Composting is a biological process in which (a) 1 only
Microorganisms decompose the organic (b) 2 only
matter.
(c) Both 1 and 2
3. Oil cakes are insoluble in water but their
(d) Neither 1 nor 2
nitrogen takes a long time to absorb in
plants.
Which of the statements given above is/are Q23. Which among the following crops can be
correct? associated with the 'Ratoon cropping'
(a) 1 and 2 only method?
(b) 2 only 1. Banana
(c) 2 and 3 only 2. Sugarcane
(d) 1, 2 and 3 3. Sorghum
4. Pineapple
Q21. With reference to Horticulture crop, consider Select the correct answer using codes given
the following statements:
below:
1. Horticulture and market gardening is well
(a) 1, 2 and 3 only
developed in densely populated industrial
districts of African countries and the (b) 1 and 2 only
Mediterranean regions. (c) 2, 3 and 4 only
2. The regions where farmers specialize in (d) 1, 2, 3 and 4
vegetables only, farming is known as truck
farming.
Which of the statements given above is/are Q24. Consider the following statements regarding
correct? tillage in India:
(a) 1 only 1. Tillage is beneficial to reduce runoff of water
and reduce soil erosion.
(b) 2 only
2. Zero Tillage degrades organic matter content
(c) Both 1 and 2 due to high mineralisation of Soil.
(d) Neither 1 nor 2 Which of the statements given above is/are
correct?

Q22. With reference to the agriculture in India, (a) 1 only


consider the following statements: (b) 2 only
1. Sugarcane and Sugar industry is India’s
(c) Both 1 and 2
largest agro-based industry, before the
cotton industry. (d) Neither 1 nor 2
2. States announce State Advised Price (SAP)
at levels higher than FRP is one of the factors
Q25. Consider the following statements regarding
of increased production.
Minimum Support Price (MSP):

97
1. The MSPs are announced during harvesting 4. It aims to minimize symbiotic and
season for certain crops on the basis of the synergistic relationships between site
recommendations of the Commission for components.
Agricultural Costs and Prices (CACP). Which of the statements given above is/ are
2. The MSPs are fixed at incentive levels to correct?
induce more investment by farmers in the (a) 1, 2 and 3 only
farm sector.
(b) 1 and 2 only
Which of the statements given above is/are (c) 2, 3 and 4 only
incorrect?
(d) 3 and 4 only
(a) 1 only
(b) 2 only
Q28. Consider the following statements:
(c) Both 1 and 2
1. Sodium, Iron and Manganese in large
(d) Neither 1 nor 2 amounts play a significant role in plant
growth.

Q26. Which of the following are microorganisms 2. Nitrogen is helpful in photosynthesis and an
acting as Bio-fertilizers? integral part of chlorophyll.

1. Rhizobium 3. Potassium gives an Energy to plants for


water to flow in upward direction till leaves.
2. Anabaena
Which of the statements given above is/are
3. Sulphur dissolving bacteria correct?
4. Fish meal (a) 1, 2 and 3
Select the correct answer using the code given (b) 2 and 3 only
below:
(c) 3 only
(a) 1 and 3 only
(d) 1 and 2 only
(b) 2 and 4 only

(c) 1, 2 and 4 only


Q29. Consider the following statements about
(d) 1, 2, 3 and 4 agricultural subsidies in WTO:
1. The agricultural subsidies which cause
Q27. Which of the following are the characteristics minimal or no distortions to trade are put
under the Blue box.
features of Permaculture:
1. It can help degraded ecosystems recover 2. Amber Box includes government policies of
health and wilderness. minimum support prices (as MSP in Indiafor
agricultural products.
2. It promotes organic agriculture and
completely eliminates use of pesticides. Which of the statements given above is/are
correct?
3. It can be applied in any ecosystem, no matter
how degraded. (a) 1 only
(b) 2 only
(c) Both 1 and 2

98
(d) Neither 1 nor 2

Answer Key
1. Ans: D 16. Ans: D
2. Ans: B 17. Ans: C
3. Ans: D 18. Ans: B
4. Ans: A 19. Ans: A
5. Ans: B 20. Ans: A
6. Ans: A 21. Ans: B
7. Ans: A 22. Ans: B
8. Ans: C 23. Ans: D
9. Ans: D 24. Ans: A
10. Ans: A 25. Ans: A
11. Ans: B 26. Ans: D
12. Ans: B 27. Ans: A
13. Ans: A 28. Ans: B
14. Ans: B 29. Ans: B
15. Ans: A

99
Mineral resources
Q1. Consider the following Minerals: 1. Diamonds fields exist in more than one state
1. Manganese in India.
2. Nickel 2. Commercial diamond mining is done in
more than one state in India.
3. Chromite
3. Till recently, the Diamond cutting industry
4. Dolomite
in India was dependent on imports of uncut
5. Bauxite diamonds, but India has now become self-
6. Mica sufficient in Diamond production.
Which of the above minerals are metallic in Which of the above statements is/are correct?
nature? (a) 1 only
(a) 1,2,4 and 5 only (b) 1 and 2 only
(b) 1,2,3 and 5 only (c) 2 and 3 only
(c) 2,3,4 and 6 only (d) 1 and 3
(d) 1,3,4,5 and 6 only
Q5. With reference to Monazite, consider the
Q2. ‘Limonite, ‘Siderite’ and ‘Lodestone’, following statements:
sometimes mentioned in the context of mining 1. In India, Monazite is found in beach sand
in India, are inferior ores of: only.
(a) Copper 2. In India, Monazite is exploited for extraction
(b) Manganese of rare earths and thorium.
(c) Iron 3. Tamil Nadu and Andhra Pradesh have
multiple deposits of monazite
(d) Bauxite
4. Odisha has no monazite deposits.
Which of the above statements are correct?
Q3. Which of the following minerals are found
naturally in the State of Odisha? (a) 1,2 and 4 only
1. Iron ore (b) 2 and 3 only
2. Manganese (c) 1 and 3 only
3. Chromium (d) 2,3 and 4 only
4. Bauxite
Choose the correct answer using the code given Q6. Consider the following statements:
below: 1. Cuddapah is known for its Limestone
(a) 1 and 2 only deposits
(b) 2, 3 and 4 only 2. Subarnarekha River is known for its gold
placer deposits.
(c) 1, 3 and 4 only
3. The Maikal Range is known for its Bauxite
(d) 1, 2, 3 and 4
deposits.
Which of the above statements are correct?
Q4. Consider the following statements:
(a) 1 and 2 only

100
(b) 2 and 3 only Which of the above categories of feed stock can be
(c) 1 and 3 only used for production of ethanol under the Ethanol
(d) 1,2 and 3 Blending Programme of the Ministry of Petroleum

Q7. Consider the following statements: and Natural Gas?

1. Majority of lignite deposits in India occur in (a) 1,2,4 and 6 only


the Chotanagpur plateau. (b) 1,3,4,5 and 6 only
2. There is no trace of anthracite coal deposits (c) 2,3 and 5 only
in India. (d) 1,2,3,4,5 and 6
3. Peat deposits have been found in the Nilgiri
hills of Western Ghats.
Q10. Which of the following industries can be
4. The Gondwana Coal has traces of sulphur, categorised as Footloose industries?
but is completely free of moisture.
1. Mobile and Computer manufacturing
Which of the above statements is/are correct?
2. Diamond Cutting
(a) 3 only
3. Aluminium Smelting
(b) 1 and 3 only
4. Fertilizers Industry
(c) 3 and 4 only
5. Automobile Manufacturing
(d) 1,2 and 4 only
Select the correct answer using the codes given
below
Q8. In which of the following regions has The (a) 1,2 and 5 only
Ministry of Petroleum and Natural Gas not
(b) 1,3 and 4 only
found prospective shale gas reserves in India?
(c) 2,3 and 4 only
(a) Gangetic Basin
(d) 1,2,3,4 and 5
(b) Assam-Arakan Basin
(c) Cauvery Basin
Q11. With reference to Sugar industry in India,
(d) Bombay High Field
consider the following statements:
1. While sugar mills are generally located close
Q9. Consider the following: to sugarcane growing areas, sugar refineries
1. B-heavy Molasses can be set up at relatively distant locations.

2. C-heavy Molasses 2. In India, the pricing of sugarcane is


3. Surplus rice with Food Corporation of India governed by the statutory provisions of the
(FCI) Sugarcane (ControlOrder, 1966 issued under
the Essential Commodities Act, 1955.
4. Damaged food grain
Which of the above statements is/are
5. Maize
incorrect?
6. Sugarcane juice
(a) 1 only

101
(b) 2 only 3. In India, RORO services are provided via
railways only.
(c) Both 1 and 2
Which of the above statements is/are correct?
(d) None of the above
(a) 2 only
(b) 1 and 2 only
Q12. Which of the following industries can be
(c) 1 and 3 only
categorised as Sunrise industries?
(d) 1,2 and 3
1. Hybrid Electric Vehicles
2. Green Hydrogen
Q15. Cotton Industry in India is one of the oldest
3. Genome Sequencing
industries, with reference to this Consider the
4. Thermal Power Plants following statements:
5. 5G Mobile Network 1. After the partition, the major cotton textile
6. Technical Textiles industry went to Pakistan, only 29 percent
left with India.
Select the correct answer using the code given
below: 2. At present, the handlooms on the
decentralized sector produces more than the
(a) 1,2,3 and 5 only
powerloom sector.
(b) 1,4,5 and 6 only
3. The Swadeshi Movement gave a major
(c) 1,2,3,5 and 6 only impetus to the cotton textile industry.
(d) 1,2,3,4,5 and 6 Which of the statements given above are
correct?
Q13. “This pipeline runs through the states of (a) 1 and 2 only
Gujarat, Madhya Pradesh and Uttar (b) 2 and 3 only
Pradesh.It has been constructed by the Gas
(c) 1 and 3 only
Authority of India Limited and is 1750
kilometres long. It connects the Bombay High (d) 1, 2 and 3
with Northern India.”
Which of the following pipelines in India is best Q16. Consider the following statements:
described by the above excerpt?
1. The distance between rails is one metre is
(a) Salaya-Koyali-Mathura Pipeline known as Broad Gauge.
(b) Hajira-Bijapur-Jagdishpur Gas Pipeline 2. The distance between the rails is 0.762
(c) Jamnagar-Loni LPG Pipeline metre or 0.610 metre, known as Narrow
(d) Kandla-Bhatinda Pipeline gauge.
3. Narrow gauge is confined to hilly areas of
the country.
Q14. Consider the following statements:
Which of the statements given above are correct?
1. The railways’ share in transportation of
surface freight in India has declined since (a) 1 and 2 only
independence. (b) 2 and 3 only
2. Rail transportation is more energy efficient (c) 1 and 3 only
than road transport.

102
(d) 1, 2 and 3 1. Copper found in all three kinds of rocks and
brown and red coloured copper used in
electrical activities.
2. Copper mixed with zinc alloy is known as
bronze and with tin as brass.
3. Rajasthan has the largest deposits of copper-
Q17. With reference to Waterway Transport, ore and Madhya Pradesh is the largest
Consider the following statements producer from quantity point of view.
1. India has 14,500 km of navigable Which of the statements given above are correct?
waterways, contributing about 1% to the
(a) 1 and 2 only
country’s transportation comprising rivers,
canals, backwaters, creeks, etc. (b) 2 and 3 only

2. Approximately 95 per cent of India’s foreign (c) 1 and 3 only


trade by volume and 70 per cent by value (d) 1, 2 and 3
moves through ocean routes.
Which of the statements given above is/are correct? Q20. Consider the following statements:
(a) 1 only 1. Bauxite is an igneous rock found in the
(b) 2 only northwest part of India.
(c) Both 1 and 2 2. Bauxite is used in the manufacturing of
(d) Neither 1 nor 2 Aluminium.
3. Tamil Nādu, Karnataka and Goa are major
producers of bauxite.
Q18. Consider the following statements regarding
Oil and Gas pipeline Which of the statements given above is/are
incorrect?
1. Solids can be transported by pipelines after
converting them into slurry. (a) 3 only

2. Asia’s first cross country pipeline covering (b) 1 and 3 only


was constructed by OIL from Naharkatia (c) 2 only
oilfield in Assam to Barauni refinery in (d) 1, 2 and 3
Bihar.
Which of the statements given above is/are
Q21. With the reference to Major mineral, consider
incorrect?
the following statements:
(a) 1 only
1. Feldspar has light cream to salmon pink
(b) 2 only colour used in ceramics and glass making.
(c) Both 1 and 2 2. Quartz is one of the most important
(d) Neither 1 nor 2 components of sand and granite.
3. Mica comprises of potassium, Aluminium,
magnesium, iron, silica andforms 20 per cent
Q19. With the reference to mineral resources of of the earth’s crust.
India, consider the following statements:
Which of the statements given above is/are correct?

103
(a) 1 and 2 only
(b) 2 and 3 only
(c) 1 only
(d) 1, 2 and 3
Answer Key
Q22. Which of the following is/are non- metallic
1. Ans: B 13. Ans: B
minerals?
2. Ans: C 14. Ans: B
1. Dolomite 3. Ans: D 15. Ans: C
4. Ans: A 16. Ans: B
2. Atomic mineral
5. Ans: B 17. Ans: C
3. Salt 6. Ans: D 18. Ans: D
7. Ans: A 19. Ans: C
4. Zinc
8. Ans: D 20. Ans: B
Select the correct answer using the code given 9. Ans: D 21. Ans: A
below. 10. Ans: A 22. Ans: C
11. Ans: D 23. Ans: B
(a) 2 and 3 only
12. Ans: C
(b) 2, 3 and 4 only
(c) 1 and 3 only
(d) All of the above

Q23. With the reference to Fuel cell, consider the


following statements:
1. Hydrogen can be used in fuel cells to
generate power using a combustion rather
than chemical reaction.
2. Fuel cells can supply combined heat and
power to Military installations at remote
locations.
3. Fuel cells can increase efficiency and
facilitate faster refueling of electric
vehicles.
Which of the statements given above is/are
correct?
(a) 1 only
(b) 2 and 3 only
(c) 1 and 2 only
(d) 1, 2, and 3

104
Miscellaneous
Q1. With the reference to the Major minerals, Q3. With the reference to the composition of the
consider the following statements: atmosphere, consider the following
1. Half of the earth's crust is composed of statements:
feldspar. 1. Carbon dioxide is transparent to the
2. Quartz consists of silica and used in radio incoming solar radiation but opaque to the
and Radar. outgoing terrestrial radiation.

3. Mica is commonly found in igneous and 2. Water vapor absorbs parts of the isolation
metamorphic rocks. from the sun and preserves the earth's
radiated heat and increases from the equator
Which of the statements given above are correct?
towards the poles
(a) 1 and 2 only
3. Dust and salt particles act as hygroscopic
(b) 2 and 3 only nuclei around which water vapour
(c) 1 and 3 only condensed to produce clouds.
(d) 1, 2 and 3 Which of the statements given above is/are correct?
(a) 1 and 2 only

Q2. Consider the following statements regarding (b) 1 and 3 only


to structure of the atmosphere: (c) 2 and 3 only
1. Height of the troposphere is average but (d) 1, 2 and 3
increases at the pole and decreases at the
equator.
Q4. With reference to the Indus River, consider
2. The temperature in the troposphere
the following statements:
decreases at the rate of 1°C for every 165m
of height. 1. Indus is the easternmost river of the
Himalayan rivers in India.
3. The zone separating the troposphere from
the stratosphere is known as the stratopause. 2. It enters into Pakistan near Chilasin the
Dardistan region.
Which of the statements given above is/ are
incorrect? 3. The Jhelum is the largest tributary of the
Indus.
(a) 2 only
Which of the statements given above is/are
(b) 1 and 3 only
incorrect?
(c) 1 and 2 only
(a) 1 and 3 only
(d) All of the above
(b) 2 only

(c) 1 and 2 only

(d) All of the above

105
2. Mass movement is the movement of
weathered materials down a slope due to
gravitational force.
Which of the statements given above is/are correct?
Q5. Which of the following National parks are
situated in Nilgiri Biosphere reserves? (a) 1 only

1. Nagarhole National Park (b) 2 only

2. Eravikulam National Park (c) Both 1 and 2

3. Silent Valley National Park (d) Neither 1 nor 2

4. Mukurthi National Park


Select the correct answer using the code given Q8. Consider the following pairs about lake
below: formation and associated reason behind it:

(a) 1 and 2 only Lake’s formation Reasons

(b) 2, 3 and 4 only 1. Caldera lakes- Forms by Erosion

(c) 1, 3 and 4 only 2. Karst lakes- Volcanic activity

(d) 1, 2, 3 and 4 3. Cirque lakes- Glaciation


Which of the pairs given above is/are correctly
matched?
Q6. Consider the following statements:
(a) 1 only
1. The Kashmir Himalayas are famous for
Karewa, which are useful for the cultivation (b) 1 and 3 only
of Zafran. (c) 3 only
2. ZojiLa pass on the Great Himalayas, Banihal (d) 1, 2 and 3
pass on the Zaskar, Photu La pass on the pir
panjal found in Northwestern Himalayas.
3. Pangong Tso and Tso Moriri are found in the
Northwestern Himalayas. Answer Key

Which of the statements given above is/are


1. Ans: D
correct?
2. Ans: B
(a) 1 only 3. Ans: B
4. Ans: A
(b) 2 and 3 only
5. Ans: C
(c) 1 and 3 only 6. Ans: C
7. Ans: B
(d) 1, 2 and 3 8. Ans: C

Q7. Consider the following statements:


1. Weathering is active wearing away from the
earth's surface by moving agents like
running water, wind, ice and waves.


106

You might also like